Simulacro cto

November 9, 2017 | Author: fany1217 | Category: Menstrual Cycle, Pregnancy, Childbirth, Luteinizing Hormone, Diabetes Mellitus
Share Embed Donate


Short Description

examen cto...

Description

booksmedicos.org

SIMULACRO 5

1.

Un paciente varón de 73 años de edad presenta un episodio depresivo mayor desde hace unos 3 meses; hace 7 años tuvo un problema similar y recibió TEC, mejorando por completo; después mantuvo varios años tratamiento con venlafaxina a 150 mg, que abandonó hace 3 años, permaneciendo asintomático hasta el inicio del episodio actual; en esta ocasión su médico de cabecera le prescribió inicialmente sertralina a 100 mg pero, ante la ausencia de mejoría tras 6 semanas de tratamiento, cambió a venlafaxina, sin obtener respuesta a pesar de alcanzar dosis elevadas y mantenerlo cerca de 2 meses; ¿cuál de las opciones siguientes parece menos acertada?

1. Lo más probable es que se trate de una crisis de angustia. 2. La normalidad de la exploración realizada no excluye todas las causas médicas de este síndrome. 3. Con los datos recogidos no se puede realizar el diagnóstico de trastorno de angustia. 4. Es fundamental explicar a la paciente las características de su problema para intentar prevenir complicaciones psicológicas. 5. La ausencia de factores estresantes nos hará dudar del origen psiquiátrico del problema.

3.

1. Puede plantearse la combinación de venlafaxina y mirtazapina. 2. El litio se ha utilizado en depresión resistente, añadido al antidepresivo. 3. Probablemente la TEC sea la alternativa más eficaz, vistos los antecedentes. 4. Los estimulantes anfetamínicos pueden ayudar a los pacientes en donde predomine la anergia. 5. La combinación de IMAOs y venlafaxina consigue resultados espectaculares en pacientes con respuestas parciales.

2.

Acude a Urgencias del Centro de Salud una mujer de 21 años de edad que hace 30 minutos comenzó a notar disnea progresiva, acompañada de dolor precordial, náuseas, palpitaciones y sensación de inestabilidad; se realiza ECG en donde sólo se detecta taquicardia sinusal a 140 lpm; la auscultación cardiopulmonar no evidencia anomalías; la glucemia capilar es de 90 mg/dL. Carece de antecedentes médicos relevantes. Al cabo de 15-20 minutos los síntomas han cedido por completo, dejando un cansancio extremo; en ese momento la paciente comienza a llorar asustada por lo que le acaba de pasar. Niega preocupaciones especiales en su vida y no encuentra una causa para explicar lo sucedido. En relación con este problema de salud, ¿cuál de las siguientes es INCORRECTA?

Un antiguo compañero de carrera te comenta que “de siempre” ha tenido grandes dificultades para exponer trabajos en público pues le preocupa mucho lo que vayan a pensar de él los que le escuchen; cuando le toca hacerlo comienza a temblar, suda profusamente y tartamudea; por estos motivos intenta evitar estas situaciones siempre que puede. Lamentablemente durante la residencia va a tener que presentar sesiones clínicas en su Servicio casi cada mes. En otros contextos no presenta estas dificultades con la gente. ¿Cuál de las siguientes afirmaciones es CORRECTA? 1. Esto le sucede a la mayoría de la gente, por lo que no debe preocuparse. 2. Probablemente se trate de una “fobia social específica”. 3. Debería consultar con un psicoanalista para mejorar sus habilidades sociales. 4. Podría tomar paroxetina de forma puntual para aliviar sus síntomas en el caso de que tuviera que presentar una sesión. 5. No se ha demostrado que estos pacientes abusen del alcohol de forma significativa.

4.

Señale el trastorno mental para el que NO está indicado el uso de escitalopram: 1. Episodio depresivo mayor. 2. Dolor neuropático en diabéticos. 3. Prevención de ataques de pánico.

booksmedicos.org 4. Fobia social grave. 5. Trastorno obsesivo-compulsivo.

5.

8.

En relación con el uso de clozapina como antipsicótico una de las siguientes afirmaciones es INCORRECTA: 1. Se utiliza como fármaco de reserva en pacientes resistentes a otros antipsicóticos. 2. Se relaciona con un importante aumento de peso a medio-largo plazo. 3. Se recomienda la realización de controles hematológicos por el riesgo de agranulocitosis. 4. A dosis elevadas reduce el umbral convulsivo y debería asociarse un antiepiléptico. 5. Tiene importantes efectos anticolinérgicos y se han comunicado casos de íleo.

6.

7.

Un paciente varón, de 72 años de edad, ha sido operado anteayer de un adenocarcinoma de próstata mediante una resección transuretral; en la noche pasada protagonizó un episodio de agitación psicomotriz, arrancándose la sonda uretral e intentando escapar de la habitación; gritaba que estaba secuestrado y decía que estaba rodeado de zombies que querían devorarle; fue preciso administrarle un fármaco por vía parenteral y ahora se encuentra adormilado, con un nivel de conciencia bajo. En relación con el problema que presenta, ¿cuál de las siguientes afirmaciones es CORRECTA? 1. La existencia de alucinaciones visuales orienta hacia un origen tóxico del cuadro. 2. Lo habitual es que estos pacientes presenten un aumento de su capacidad atencional. 3. Probablemente le hayan administrado un neuroléptico sedante para controlar la agitación. 4. En el EEG puede encontrarse un enlentecimiento generalizado. 5. La presencia de esta complicación no modifica el pronóstico del postoperatorio.

Una compañera de carrera tiene la necesidad constante de que los demás se ocupen de ella y la ayuden, incluso en las decisiones menos importantes; por eso tiene una actitud exageradamente sumisa con los demás, manifestando con frecuencia un servilismo inquebrantable hacia sus amistades y un enorme miedo de verse sola. ¿Qué tipo de trastorno de la personalidad presenta? 1. 2. 3. 4. 5.

SIMULACRO 5

9.

Histriónica. Dependiente. Límite. Evitativa. Paranoide.

Una mujer de 43 años de edad con antecedentes de tratamiento por un síndrome depresivo es encontrada en su dormitorio en estado de coma; presenta pupilas midriáticas, no reactivas, y una respiración superficial; en el ECG se evidencia un QTc prolongado; durante el traslado al hospital sufre una crisis convulsiva generalizada tónico-clónica. ¿Qué medida de las siguientes es INADECUADA ante la sospecha diagnóstica más probable?

Una mujer de 43 años de edad se quedó atrapada en un ascensor durante toda una noche al terminar su jornada laboral; tras ser rescatada muestra un rechazo absoluto a volver a usar ese tipo de aparatos; en alguna ocasión que lo ha intentado ha presentado un episodio agudo de intensa ansiedad; no presenta otros síntomas pero se encuentra molesta ante lo que le sucede dado que la oficina en la que trabaja se encuentra en los pisos más altos de un edificio de gran altura, por lo que se encuentra de baja laboral. ¿Qué diagnóstico emitiríamos en este caso? 1. 2. 3. 4. 5.

10. 1. Es imprescindible la administración de bicarbonato para reducir la cardiotoxicidad. 2. Puede recurrirse a la procainamida para tratar las arritmias ventriculares. 3. No es raro que se produzca una hipotensión refractaria. 4. La monitorización electrocardiográfica predice el pronóstico de estos pacientes. 5. No disponemos de un antídoto específico para este tipo de fármacos.

Fobia específica. Claustrofobia. Trastorno por estrés postraumático. Simulación. Trastorno por estrés agudo.

¿En cuál de los siguientes casos NO se consideraría necesario el ingreso hospitalario para lograr la desintoxicación alcohólica? 1. Antecedentes de convulsiones asociadas a la abstinencia. 2. Presencia de cirrosis hepática avanzada. 3. Ingesta habitual de grandes cantidades de alcohol. 4. Paciente que carece de domicilio estable. 5. Patrón de consumo “mediterráneo”.

-2-

booksmedicos.org 11.

Durante el seguimiento de una paciente joven, afectada de una enfermedad inflamatoria intestinal crónica, notas cómo van surgiendo en ti unos sentimientos de afecto hacia ella; movido por los mismos decides no someterla en la última revisión a la colonoscopia de control protocolizada, dado que ella vive dichas exploraciones de forma muy negativa. ¿Qué mecanismo psicológico está determinando tus decisiones? 1. 2. 3. 4. 5.

12.

5. Al final de la fase lútea se produce un descenso de FSH previo a la menstruación.

14.

Respecto a las amenorreas primarias, es FALSO que: 1. La causa más frecuente son las disgenesias gonadales. 2. El síndrome de Swyer es una disgenesia gonadal en la que es frecuente la aparición de talla baja y malformaciones asociadas. 3. El síndrome de Rokitansky en la que suele existir alteraciones uterinas y de los dos tercios superiores de vagina. 4. El síndrome de Morris tiene cariotipo 46XY y puede desarrollar disgerminoma. 5. En la hiperplasia suprarrenal congénita aparecen genitales externos masculinizados, debido a un trastorno enzimático de la síntesis del cortisol.

Anulación. Contratransferencia. Formación reactiva. Despersonalización. Sublimación.

Don Rafael es un varón de 65 años de edad que hace tres años se jubiló tras toda una vida trabajando como contable en una entidad financiera; progresivamente ha ido mostrándose apático y desinteresado hacia su entorno, descuidando su aspecto físico y protagonizando algunos incidentes extraños; por ejemplo, ha sido descubierto en varias ocasiones robando chucherías o pasteles, o comiéndolos directamente del mostrador; no parece preocupado por esto y de hecho no refiere queja ninguna; su lenguaje es escaso y su respuesta emocional escasa. Tiene leves fallos de memoria reciente pero su funcionamiento en otras áreas es normal y en las pruebas complementarias no se encuentra causa aparente para su problema. ¿Cuál de las afirmaciones siguientes sobre su problema es INCORRECTA?

15.

Usted sabe que el síndrome del ovario poliquístico (SOP) se caracteriza por una alteración hormonal compleja. Respecto a la fisiopatología, es FALSO que: 1. Afecta al 4-10% de las mujeres en edad reproductiva. 2. Aparece una hiperpulsatilidad de la GnRH. 3. Existe una concentración sérica media de LH elevada, con FSH normal o en el límite inferior de la normalidad. 4. En un 20-30% de casos aparece hiperprolactinemia moderada, que alcanza niveles anovulatorios. 5. El DHEA-S se encuentra elevado en el 20% de las pacientes con ritmos circadianos de ACTH y cortisol normales.

1. En algunos pacientes se encuentran inclusiones citoplasmáticas redondas, intensamente argirófilas, en neuronas corticales e hipocampales. 2. Las técnicas de neuroimagen estructural pueden evidenciar atrofia frontal y temporal. 3. Es frecuente que se encuentren reflejos primitivos en la exploración neurológica. 4. Las alucinaciones visuales complejas son típicas de estos pacientes. 5. No es raro que estos pacientes sean calificados inicialmente como depresivos.

13.

SIMULACRO 5

16.

En relación al ciclo genital femenino es FALSO que: 1. La GnRH sintetizada por el hipotálamo estimula la síntesis de gonadotropinas hipofisarias. 2. En la primera fase del ciclo, la FSH produce un aumento del recuento folicular. 3. La LH tiene un solo pico, previo a la ovulación. 4. El cuerpo lúteo produce progesterona y estrógenos.

-3-

Paciente de 47 años que acude a urgencias porque desde hace 15 días presenta un sangrado que comenzó como una regla normal, pero que no ha cedido, incluso llegando a sangrar más que una regla. Como antecedentes ginecológicos destacan 2 partos eutócicos y un legrado por aborto diferido; reglas cada 26 días, de 5 días de duración, no dolorosas. Le solicita un test de embarazo en orina, siendo negativo su resultado. A la exploración se objetiva un cérvix de multípara, con restos hemáticos en vagina en cantidad similar a una regla. Le realiza una ecografía transvaginal en la que objetiva un útero regular en anteversión de 8 cm, con un endometrio de aspecto secretor de 13 mm sin que parezca observarse patología. Anejo derecho normal, anejo izquierdo con formación econegativa de 38 mm en ovario izquierdo de aspecto benigno. Respecto al cuadro clínico que presenta la paciente, es FALSO que:

booksmedicos.org 1. Para cohibir la hemorragia, se debe pautar tratamiento estrogénico. 2. Es recomendable el uso de AINEs y preparados con hierro oral. 3. El cuadro clínico puede explicarse a ciclos disovulatorios. 4. Si persisten los cuadros de sangrado, se puede plantear el uso de anticonceptivos hormonales, DIU de levonorgestrel o cirugía. 5. Es recomendable repetir la ecografía a dicha paciente en 3 meses.

17.

4. Es más frecuente en la raza negra. 5. Provoca esterilidad por alteración de la cavidad uterina.

20.

21.

18.

Inseminación artificial de cónyuge. Inseminación artificial de donante. Coito programado. Fecundación in vitro. Fecundación in vitro con donación de ovocitos.

Candida albicans. Candida glabrata. Virus del papiloma humano. Trichomona vaginalis. Gardnerella vaginalis.

¿Qué tratamiento de elección recibiría una mujer de 29 años, con dolor abdominal, dolor anexial, temperatura de 38.2º C, que en los últimos meses ha mantenido relaciones sexuales sin protección, presenta leucocitosis en la analítica y en la ecografía se observa en el anejo derecho una imagen de 5x7 cm con el signo de la “rueda dentada”? 1. Cirugía laparoscópica. 2. Ceftriaxona i.m. dosis única y Doxiciclina v.o. 14 días. 3. Amoxicilina y ácido clavulánico vía oral durante 7 días. 4. Anticonceptivos orales durante 6 meses y métodos barrera la primera semana. 5. Cirugía radical: histerectomía y doble anexectomía.

Mujer de 40 años con un embarazo y parto normal hace 4 años que tras 3 laparoscopias para quistectomías ováricas por endometriosis presenta hipermenorrea y dismenorrea intensa a pesar de tratamiento con anticonceptivos y AINE junto a dolor intermenstrual que le ha obligado a pedir varias bajas laborales. Además, en los últimos 4 meses presenta disquecia los 2 primeros días de menstruación. En la ecografía transvaginal se identifican 2 quistes endometriósicos en ovario izquierdo (de 3 y 4 cm) y otro en ovario derecho (de 4 cm). ¿Qué opción terapéutica recomendaría?

22.

En relación con el liquen escleroso vulvar (LEV), señale lo falso: 1. Afecta principalmente a la mujer peri y postmenopáusica, aunque puede aparecer a cualquier edad, incluido en la infancia. 2. Se caracteriza principalmente por prurito de intensidad variable, intermitente o permanente. Aunque no es infrecuente el ardor vulvar y la dispareunia. 3. El riesgo de desarrollar un carcinoma epidermoide se estima en un 25-30% de los casos. 4. La confirmación diagnóstica se realiza mediante el examen histológico. 5. El tratamiento del LEV es médico, orientado principalmente a calmar los síntomas y evitar la evolución atrófica.

1. 2. 3. 4.

Tratamiento con análogos de GnRH. Laparotomía: histerectomía y doble anexectomía. Laparoscopia: quistectomía ovárica bilateral. Laparoscopia: liberación de adherencias peritoneales. 5. Colocación de DIU-Levonorgestrel.

19.

¿En qué infección vulvovaginal está indicado el tratamiento de la pareja sistemáticamente? 1. 2. 3. 4. 5.

Ante una paciente de 43 años sin hijos, baja reserva ovárica y con un factor tubárico, ¿qué técnica de reproducción asistida consideraría más indicada? 1. 2. 3. 4. 5.

SIMULACRO 5

Señale la afirmación verdadera con respecto a la endometriosis: 1. El tabaco es factor de riesgo. 2. El patrón menstrual típico son ciclos cortos con sangrado menstrual abundante. 3. La prevalencia se mantiene constante en un 5% de las mujeres en edad fértil.

23.

-4-

Mujer de 81 años, diabética e hipertensa, institucionalizada en una residencia de mayores, que es trasladada al servicio de urgencias del hospi-

booksmedicos.org tal, tras haber observado una cuidadora la presencia de una ulceración en región vulvar, que se acompaña de secreción serohemática y mal olor. Al preguntar a la paciente, que está consciente y orientada, refiere que “lleva mucho tiempo con esa lesión, que es dolorosa y que previamente le picaba mucho”. En cuanto a la patología que presenta la paciente, es FALSO:

26.

1. La edad de aparición más frecuente se encuentra entre los 65-80 años, siendo rara antes de los 30 años. 2. Los tratamientos con agentes inmunosupresores y el tabaco son factores de riesgo. 3. La localización más frecuente son los labios mayores, pero puede afectar también a labios menores, clítoris, horquilla y periné. 4. Son de utilidad en su diagnóstico la vulvoscopia y la tinción con azul de toluidina. 5. Se extiende principalmente por vía hematógena, siendo más rara la extensión linfática o por continuidad.

24.

Paciente de 59 años con pérdidas de orina ocasionales con los esfuerzos, no necesitando protector diario. Su calidad de vida no se ve ampliamente afectada. ¿Cómo catalogaría su incontinencia y qué tratamiento le propondría?

28.

Adenocarcinoma seroso. Teratoma. Adenoma mucinoso. Tumor de células claras. Tumor del seno endodérmico.

Mujer de 42 años cuya mamografía de screening es informada como BIRADS 4, ¿qué significa dicho hallazgo? 1. Estudio de imagen incompleto, se necesitan otras pruebas de imagen. 2. Mamografía con hallazgos probablemente benignos pero para confirmarlo es preciso hacer una nueva mamografía en 6 meses. 3. Mamografía negativa. La paciente puede continuar el programa de cribado con normalidad. 4. Mamografía con hallazgos altamente sospechosos de malignidad y, por ello, es precisa la biopsia para confirmar el diagnóstico. 5. Mamografía con hallazgos probablemente malignos por lo que es necesaria la realización de biopsia diagnóstica.

Mujer de 52 años cuya última regla fue hace más de un año, presenta una hemorragia vaginal de varios días de evolución, sin ningún otro síntoma asociado. La ecografía muestra un endometrio engrosado de 11 mm, sin áreas sólidas ni hiperrefringentes. Se le practica una histeroscopia dignóstica con el resultado de hiperplasia endometrial atípica. ¿Cuál es la conducta a seguir más adecuada? 1. 2. 3. 4. 5.

IB. II. IIIB. IIIC1. IIIC2.

Mujer de 43 años que presenta tumoración de 5 cm en ovario izquierdo, con índices de resistencia disminuidos y zona sólidas-mixtas en su interior en la ecografía. Hace 8 años fue intervenida por una masa anexial en el mismo lado, con diagnóstico anatomopatológico de endometrioma. Presenta elevación de marcadores tumorales especialmente del CA 125. ¿Cuál es la estirpe tumoral que más probablemente presente la paciente? 1. 2. 3. 4. 5.

1. IU de urgencia: colocación de banda suburetral libre de tensión. 2. IU de esfuerzo: tratamiento rehabilitador. 3. IU mixta: colocación de banda suburetral libre de tensión. 4. IU de esfuerzo: colocación de banda suburetral libre de tensión. 5. IU de urgencia: administración de fármacos anticolinérgicos.

25.

Paciente de 70 años con diagnóstico histológico de cáncer de endometrio grado 2. La resonancia magnética nos informa de una afectación de mas del 50% de la pared miometrial y posible afectación cervical. Además describen ganglios pélvicos aumentados de tamaño con alta probabilidad de invasión carcinomatosa. El resto de la pelvis no muestra afectación tumoral. ¿En qué estadio tumoral se encuentra la paciente? 1. 2. 3. 4. 5.

27.

SIMULACRO 5

Histerectomía simple. Histerectomía radical. Conducta expectante. Análogos de GnRH. Histeroscopia quirúrgica con ablación endometrial.

29.

-5-

El diagnóstico prenatal tiene como finalidad diagnosticar con la mayor precocidad posible un de-

booksmedicos.org fecto congénito o bien establecer la ausencia del mismo, ya que la confirmación de la normalidad contribuye a reducir la ansiedad materna durante el resto de la gestación. En relación a los marcadores bioquímicos de cromosomopatía para los distintos screening, es cierto que:

4. Rotura de vasa previa: reexplorar y si está en dilatación completa realizar un parto instrumental con fórceps para extracción fetal inmediata. 5. Rotura de vasa previa: cesárea urgente.

32. 1. La fracción β libre de la gonadotropina coriónica (fβ-HCG) está disminuida en la trisomía 21. 2. La proteína plasmática asociada al embarazo (PAPP-A) está aumentada en la trisomía 21. 3. La alfa feto-proteína (AFP) está disminuida en la trisomía 21. 4. El estriol no conjugado (uE3) está aumentado en la trisomía 21. 5. La inhibina A está disminuida en la trisomía 21.

30.

Paciente primigesta de 31 semanas con un embarazo gemelar monocorial biamniótico con el primer gemelo en cefálica y un peso fetal estimado de 1300 g y el segundo gemelo en cefálica con un peso fetal estimado de 1150 g, que acude a Urgencias con dinámica uterina regular y una dilatación cervical de 3 cm. La vía del parto más adecuada será: 1. Cesárea por edad gestacional inferior a 32 semanas. 2. Cesárea por pesos fetales estimados inferiores a 1500 g. 3. Cesárea por gestación gemelar monocorial. 4. Parto vaginal de ambos gemelos. 5. Parto vaginal con versión interna y gran extracción del segundo gemelo.

Acude a urgencias una paciente de 28 años con amenorrea de 9 semanas por metrorragia menor que una regla, sin dolor. Le solicita un test de embarazo que es positivo. A la exploración ginecológica se objetivan restos hemáticos escasos en vagina, no sangrado activo, cérvix cerrado. En la ecografía trasvaginal se objetiva una vesícula gestacional de 18 mm con un embrión de 3 mm en el que no se objetiva latido cardiaco. Señale la respuesta INCORRECTA:

33.

1. Se trata de una amenaza de aborto, por lo tanto trataremos con reposo relativo y control ecográfico en 7-10 días. 2. Es recomendable el uso de progesterona natural micronizada vía vaginal u oral. 3. Este cuadro suele aparecer en el 20% de las pacientes. 4. Practicaremos legrado evacuador al tratarse de un aborto diferido, ya que no hay latido. 5. La amenorrea no coincide con la datación ecográfica.

31.

SIMULACRO 5

Gestante de 33 semanas con contracciones regulares (2 contracciones /10 minutos), a la exploración cervical se objetiva un cérvix permeable un dedo, borramiento 50%, consistencia media y posición posterior. Mediante ecografía vaginal se obtiene una cervicometría de 21 mm. Analítica dentro de límites normales. En esta situación, usted indicaría: 1. Realizar test de fibronectina y sólo si es positivo realizar maduración pulmonar fetal con corticoides e hidratación. 2. Maduración pulmonar fetal con corticoides de forma domiciliaria e hidratación. 3. Maduración pulmonar fetal con corticoides en ingreso hospitalario. 4. Maduración pulmonar fetal con corticoides y tocolisis intravenosa en ingreso hospitalario. 5. Tocolisis intravenosa, hidratación y reposo absoluto en ingreso hospitalario.

Secundigesta de 38 semanas con embarazo de curso normal que tras 3 horas con dinámica uterina espontánea alcanza los 4 cm con monitorización fetal reactiva. En ese momento se produce amniorrexis espontánea con salida de líquido amniótico hemorrágico y aparece un patrón sinusoidal a 100 lpm en el registro fetal. ¿Cuál sería su diagnóstico y actitud?

34.

1. Abruptio placentae: cesárea urgente. 2. Abruptio placentae: realizar pH de sangre fetal. 3. Abruptio placentae: reexplorar y si está en dilatación completa realizar un parto instrumental con fórceps para extracción fetal inmediata.

-6-

Gestante primigesta de 36 años, que acude al hospital en la semana 39 de gestación por inicio de trabajo de parto. Después de un periodo de dilatación de 8 horas y varias exploraciones iguales sin cambios, llega a 8 cm de dilatación y la presentación se encuentra en un II plano de Hodge. El RCTG es rigurosamente normal y presenta buena dinámica uterina. Ante la no evolución y el cansancio materno, ¿qué actitud tomaría?

booksmedicos.org 1. 2. 3. 4. 5.

35.

Realizar una ventosa. Realizar una cesárea. Realizar un fórceps. Realizar una microtoma fetal (pHmetría). Dejar evolucionar 6 horas más para intentar un parto por vía vaginal.

1. No se han calculado correctamente los requerimientos calóricos, ya que la ecuación de Harris-Benedict no tiene ninguna utilidad en el medio hospitalario. 2. Deberían haber trascurrido al menos 7-10 días para que estuviese indicado iniciar nutrición parenteral. 3. Un residuo gástrico elevado puede ser una de las complicaciones que obliguen a suspender la nutrición parenteral. 4. No está indicado soporte nutricional en este paciente por tener una albúmina dentro de la normalidad. 5. Ninguna opción es correcta.

Paciente de 27 años que acude a urgencias refiriendo malestar general y temperatura de 38.3º C en domicilio. Tuvo un parto instrumental (fórceps) hace 5 días. Hace dos días fue dada de alta con buen estado general. Actualmente está con lactancia materna. La temperatura en urgencias es de 39.2 ºC. Se realiza una correcta anamnesis y una exploración minuciosa, sin hallazgos llamativos. Teniendo en cuenta el diagnóstico de sospecha, ¿qué actitud NO estaría indicada?

38.

1. Extraer analítica completa con hemograma, bioquímica, coagulación, PCR y VSG. 2. Extraer urocultivo. 3. Extraer hemocultivos. 4. Iniciar tratamiento antibiótico empírico. 5. Suspender la lactancia.

36.

En el caso anterior, a los 2 días de iniciar la nutrición el paciente comienza con disnea de reposo, y analítica con fósforo: 1.8 mEq/L. Señale la opción VERDADERA: 1. Es la complicación más grave y frecuente de la nutrición artificial. 2. Se podría haber evitado administrando al paciente un mayor aporte calórico. 3. Se suele asociar con alteraciones electrolíticas, tales con hipopotasemia, hiperfosfatemia, hipermagnesemia. 4. Entre sus posibles complicaciones hay que tener en cuenta el desencadenamiento de arritmias, con potencial mortal. 5. Sólo se asocia a la administración de nutrición parenteral.

Gestante de 31 semanas que acude a urgencias hospitalarias por presentar una tensión arterial de 150/95. No refiere tensiones altas en controles previos. Las tensiones durante su estancia son normales. Se realiza analítica de orina con resultado de normalidad. La paciente refiere cefalea sin otra sintomatología asociada. El hemograma, la coagulación y la bioquímica completa tampoco muestran alteraciones. Ante estos hallazgos, cómo clasificaría el estado hipertensivo de la paciente y qué tratamiento propondría:

39.

Señale la afirmación FALSA en relación con el Síndrome Pluriglandular Autoinmune tipo 1: 1. Es característica la candidiasis mucocutánea. 2. Al igual que en el síndrome glandular autoinmune tipo 2 se asocia a ciertos haplotipos. 3. Puede llegar a asociarse a diabetes mellitus. 4. La mayoría de los pacientes presentan adrenalitis autoinmunitaria. 5. Las mujeres presentan con mayor frecuencia hipogonadismo que los hombres.

1. Preeclampsia e ingreso urgente, dada la cefalea de la paciente. 2. Hipertensión crónica y tratamiento antihipertensivo. 3. Hipertensión gestacional y tratamiento antihipertensivo. 4. Hipertensión gestacional y controles ambulatorios de tensión arterial. 5. Preeclampsia y tratamiento antihipertensivo. 40. 37.

SIMULACRO 5

Paciente de 74 años que ingresa por anorexia, vómitos incoercibles y dolor abdominal, con pérdida de 12 kg en el último mes. En el TAC se objetiva obstrucción intestinal. Proteínas totales: 6.5 mg/ dl, albumina: 3.6 mg/dl. Se decide nutrición parenteral, aportándole 1800 kcal según la ecuación de Harris- Benedict. Señale la opción VERDADERA:

Varón de 39 años, con diarrea de semanas de evolución, de más de 10 deposiciones líquidas al día. Glucosa: 189 mg/dl, Cr: 2,1 mg/dl, K: 2,9 mEq/L, 5-HIAAo en rango de normalidad, hipoclorhidria. Señale el diagnóstico de mayor sospecha: 1. Glucagonoma. 2. Tumor carcinoide.

-7-

booksmedicos.org 3. Síndrome de Zollinger-Ellison. 4. Síndrome de Werner-Morrison. 5. Neoplasia endocrina múltiple 2B.

41.

44.

Niña de 6 años de edad con botón mamario y vello púbico. Tanner 2, velocidad de crecimiento acelerada, edad ósea 9 años y LH y FSH elevadas, con 17-OH progesterona y estradiol elevados. Respecto a la entidad que usted sospecha señale la opción que considere más correcta:

Varón de 56 años con poliuria y polidipsia y pérdida de 8 kg de peso. Glucemia venosa al azar de 324 mg/dl y cuerpos cetónicos negativos. ¿Cuál sería la actitud más correcta?

45.

Señale la opción FALSA en relación a la diabetes insípida: 1. La deshidratación severa es una manifestación clínica infrecuente en adultos sanos con diabetes insípida. 2. Una osmolalidad urinaria de 800 mOsm/kg tras la prueba de la sed es sugestiva de polidipsia primaria. 3. El síndrome de Wolfram se asocia a la existencia de diabetes insípida nefrogénica. 4. La hipoosmolalidad urinaria en relación a la plasmática tras la administración de desmopresina, es característica de la diabetes insípida nefrogénica. 5. Las manifestaciones clínicas suelen aparecer de forma más aguda en la diabetes insípida central que en la nefrogénica.

1. Insistir en la pérdida de peso y añadir un antidiabético oral como el exenatide que le ayude a perder peso. 2. Tendría que confirmar el diagnóstico de diabetes con una segunda analítica. 3. Dieta, ejercicio y comenzar tratamiento con metformina. 4. Actualmente estaría indicado iniciar tratamiento con insulina. 5. Metformina y sulfonilurea.

43.

Una paciente de 35 años, que recibe tratamiento con neurolépticos por un cuadro psicótico, acude a consultas de endocrinología por amenorrea secundaria de 6 meses de evolución. Presenta en la analítica una cifra de prolactina de 138 ng/mL, test de embarazo negativo, y en la RMN de silla turca no se observan hallazgos significativos. Señale la opción CORRECTA: 1. Ante la presencia de amenorrea se debe iniciar tratamiento con cabergolina en dosis altas. 2. La RMN normal descarta la existencia de un microprolactinoma. 3. Se debe suspender el neuroléptico y repetir el análisis en 4 semanas para valorar si la hiperprolactinemia es secundaria a fármacos. 4. Se debe consultar a su psiquiatra si es posible modificar el tratamiento, y en caso negativo, se debe iniciar tratamiento con anticonceptivos orales. 5. Se debe mantener el neuroléptico e inducir menstruaciones con progestágenos de forma cíclica.

1. Es obligatorio la realización de un TAC o RMN craneal. 2. En un alto porcentaje se debe a quistes foliculares. 3. En el caso de los niños la etiología suele ser idiopática. 4. Se trata de un cuadro de pseudopubertad precoz, ya que el desarrollo puberal no ha sido completo. 5. Debería realizarse un cariotipo.

42.

SIMULACRO 5

Mujer de 29 años, obesa (IMC 32), fumadora de 10 cigarrillos día y con antecedentes de diabetes gestacional, es remitida a tu consulta ante el hallazgo de una glucemia venosa en ayunas de 119 mg/dl confirmada en una segunda determinación (G: 117 mg/dl). ¿Qué haríamos a continuación?

46.

1. Iniciar tratamiento con metformina ya que la paciente tiene una glucemia basal alterada y otros factores de riesgo asociados. 2. Realizaría una sobrecarga oral de glucosa. 3. Iniciar tratamiento con exenatide para favorecer la pérdida de peso. 4. No hay que hacer nada. 5. Le advertiría del riesgo que tiene de diabetes y retiraría los hidratos de carbono de su dieta.

Paciente de 41 años ingresado en UCI tras síndrome coronario agudo, con ventilación mecánica y drogas vasoactivas. En una analítica se objetiva TSH: 0,23 mcUI/ml (0,4-4 mcUI/ml). ¿Cuál sería su actitud ante este hallazgo? 1. Se trata, claramente, de una enfermedad sistémica no tiroidea y, por lo tanto, no es necesario hacer nada más en este sentido. 2. Puede tratarse de una resistencia a hormonas tiroideas asociada a enfermedad severa y debemos instaurar tratamiento con dosis elevadas de L-tiroxina y corticoides. 3. Se trata de un hipertiroidismo primario, probablemente, por enfermedad de Graves-Basedow

-8-

booksmedicos.org y debemos solicitar una gammagrafía tiroidea e instaurar tratamiento con antitiroideos. 4. Se trata de un hipotiroidismo, probablemente primario, y debemos solicitar una ecografía cervical e instaurar tratamiento con L-tiroxina. 5. Puede tratarse de un síndrome de enfermedad sistémica no tiroidea por lo que ampliaremos la analítica y realizaremos un seguimiento analítico del paciente.

47.

pulmonar obstructiva crónica. Recientemente le han detectado una glicemia basal en ayunas de 143 mg/dl. Consulta a urgencias porque desde hace 24 h presenta palpitaciones rápidas e irregulares con sensación de mareo. Se realiza un electocardiograma que muestra una taquicardia irregular de QRS estrecho a 137 lpm, observándose entre los intérvalos RR múltiples pequeñas ondas de distinta duración y morfología, sin otros hallazgos. En relación a la arritmia que presenta este paciente no es cierto:

Paciente de 62 años de edad, diagnosticado hace 6 meses de carcinoma diferenciado de tiroides, tratado mediante tiroidectomía total y linfadenectomía del compartimento central, objetivándose en la anatomía patológica en diagnóstico referido con un foco de diámetro máximo de 4,5 cm y afectación ganglionar en 4 de 12 ganglios aislados. Tras el tratamiento, el paciente recibió tratamiento con 150 mCi de I-131, objetivándose en el rastreo corporal total post-dosis captación a nivel del lecho tiroideo sin otros focos a distancia. ¿Cuál de las siguientes pruebas considera menos adecuada para realizar, de entrada, en la revisión actual?

1. Esta arritmia la padece un 1-2% de la población general. 2. El riesgo de AVC cardioembólico no es despreciable. 3. Si no se controla la frecuencia cardiaca a lo largo del tiempo puede provocar insuficiencia cardiaca. 4. No tiene asociación familiar. 5. Debe realizarse el diagnóstico diferencial con en flutter auricular.

50. 1. Determinación de TSH y T4L. 2. Determinación de tiroglobulina y Ac antitiroglobulina tras estímulo con TSHrh. 3. Ecografía cervical. 4. TC cérvico-torácico con contraste. 5. Rastreo corporal total tras estímulo con TSHrh.

48.

Varón de 64 años, hipertenso y diabético tipo 2 de 8 años de evolución en tratamiento con insulina. Cardiopatía isquémica que debutó hace 2 años con ventículo izquierdo ligeramente dilatado y función sistólica del 41%, y filtrado glomerular de 39 ml/ min. El paciente recibe aspirina, atenolol y enalapril. Su ECG actual muestra bradicardia sinusal a 42 lpm, con ondas T picudas en cara anterior y un QRS de 130 m. ¿Cuál de las siguientes actitudes sería la más apropiada?

Un paciente de 17 años, varón, sin antecedentes de interés, acude al hospital por un cuadro de 2 días de evolución de fiebre alta, vómitos y dolor torácico no irradiado. Se le realiza un electrocardiograma que muestra ondas T discretamente negativas de V2 a V5 , DI, DII y aVF. Los maracdores de lesión miocárdica resultan positivos con una troponina T en meseta elevada por 8 veces su valor normal. Usted establece el diagnóstico de miocarditis. ¿ Cuál de las siguientes considera errónea de acuerdo con el diagnóstico que ha establecido? 1. Lo más fecuente es que sea causada por virus. 2. La biopsia endomiocárdica es imprescindible para el diagnóstico. 3. Los hallazgs en el ECG suelen se muy variables, desde la inversión de la onda T al ascenso del segmento ST o la aparición de un bloqueo de rama. 4. Puede presentarse en forma de muerte súbita por fibrilación ventricular. 5. Hay que realizar un ecocadiograma para valorar la función ventricular.

1. Citar al paciente en un año dado que está estable de su cardiopatía. 2. Añadir al tratamiento 1 comprimido de digoxina al día. 3. Solicitar una analítica con niveles de potasio. 4. Solicitar un angioTAC de arterias coronarias. 5. Debería implantarse un marcapasos definitivo DDD.

49.

SIMULACRO 5

51.

Paciente varón de 77 años, exfumador, con antecedentes de hipertensión arterial en tratamiento con 10 mg de enalapril; obesidad y enfermedad

-9-

Un paciente varón de 81 años, afecto de hipertensión arterial en tatamiento con diltiazem acude al servicio de Urgencias por haber presentado un síncope brusco mientras paseaba a su perro. Al interrogatorio nos explica que desde hace 4 meses, cuando sale a pasear al perro tiene que parar a los 100 metros porque presenta fatiga con sensación disneica y, en ocasiones, un dolor que describe como un peso en región retrosternal. En el ECG destacan ondas R altas en V4-V5, estando

booksmedicos.org

52.

en ritmo sinusal. A la exploración usted detecta un soplo sistólico intenso de carácter rugoso en focus aórtico irradiado a carótidas. En relación a la enfermedad que presenta el paciente, señale la corecta:

nusal con pobres voltajes y alternancia eléctrica. A la exploración física, la paciente está diaforética, oligúrica y con un pulso que disminuye con la inspiración. ¿Cuál sería su primera sospecha diagnóstica?

1. Presenta una micardiopatía hipertrófica obstructiva y hay que valorar implantar un desfibrilador automático. 2. Por la edad, cualquier intervención quirúrgica cardiaca será rechazada. 3. Precisa de una válvula en posición aórtica mecánica. 4. Precisa de una válvula en posición aórtica biológica. 5. Sin duda debería implantarse una prótesis en posición aórtica transcatéter.

1. 2. 3. 4. 5.

54.

55.

Toracocentesis. Ecocardiograma urgente-Pericardiocentesis. AngioTAC torácico urgente-Pericardiocentesis. Angiografía-fibrinolisis. Fibrobroncoscopia.

En un paciente portador de marcapasos que acude a la consulta por aumento de edemas y hepatomegalia dolorosa de 3 traveses de dedo, desde el implante del marcapasos y que niega disnea, ortopnea o DPN y a la auscultación cardiaca destaca soplo sistólico eyectivo que aumenta con la inspiración profunda, ¿qué debemos sospechar? 1. 2. 3. 4. 5.

56.

Neumotórax. Atelectasia. Taponamiento cardiaco. Neumonía. Tromboembolismo pulmonar.

En relación al caso previo, ¿cuál cree usted que sería la medida diagnóstico-terapéutica más urgente? 1. 2. 3. 4. 5.

Paciente mujer de 68 años, exfumadora, con antecedentes de hipercolesterolemia y diabetes mellitus tipo 2 de 3 años de evolución. Hace 5 meses presentó un infarto de miocardio anterolateral extenso Killip III que cursó con fibrilación venticular primaria. Fue sometida a intervencionismo coronario percutáneo, implantándose un stent farmacoactivo sobre una lesión inicialmente oclusiva en la descendente anterior proximal a las 6 horas del inicio de los síntomas. La fracción de eyección al alta hospitalaria fue del 32%, indicándose tratamiento con AAS, carvedilol, enalapril, clopidogrel y eplerenona. Usted visita a la paciente 5 meses más tarde. Se ha encontrado bien, con disnea a muy grandes esfuerzos y el ECG que le practica muestra ritmo sinusal a 72 lpm con un QRS de 90 ms. Aporta un ecocardiograma realizado la semana previa a su visita donde destaca un ventrículo izquierdo ligeramente dilatado con una función sistólica del 32%. En relación al manejo de esta paciente, ¿cuál le parece la más correcta? 1. Debe añadirse al tratamiento verapamil a dosis antianginosas. 2. Se debe indicar un desfibrilador automático implantable (DAI). 3. Se beneficiaría de un resincronizador. 4. Habrá que plantearse un trasplante cardiaco. 5. Debería añadirse al tratamiento digoxina.

53.

SIMULACRO 5

Hepatopatía crónica por virus hepatitis C. Fallo hepático agudo por amiodarona. Tromboembolismo pulmonar. Insuficiencia tricuspídea. Taponamiento cardiaco.

Varón de 78 años sin FRCV y sin enfermedades importantes que presenta cuadro de dolor torácico y síncope en la última semana. En la EF destaca soplo sistólico irradiado a carótidas que borra segundo tono y en ECG aparecen datos de HVI. ¿Cuál sería la actitud más apropiada en este paciente? 1. Ergometría. 2. Ecocardiograma. 3. Holter para descartar BAVC como causa del síncope. 4. Coronariografía por angina inestable y síncope. 5. Ergometría con isótopos por alteración en la repolarización.

Está usted trabajando en un servicio de Urgencias y atiende a una paciente con neoplasia de mama que acude por disnea. En la Rx tórax no se consigue distinguir la silueta cardiaca porque presenta un derrame pleural importante. Se ha practicado un ECG que muestra taquicardia si-

-10-

booksmedicos.org 57.

58.

Paciente de 34 años de edad, no fumador, que consulta por disnea de esfuerzo de varios años de evolución. En la exploración funcional se observa un patrón obstructivo con prueba broncodilatadora negativa y una DLCO del 56% del teórico. En la Rx tórax se aprecian signos de hiperinsuflación pulmonar y en la TAC torácica zonas de enfisema panacinar que predomina en lóbulos inferiores. Indique cuál es el diagnóstico más probable:

adenopatías mediastínicas, sin observarse captaciones patológicas a ningún otro nivel del organismo. La exploración funcional respiratoria es normal. Se le realiza una broncoscopia con biopsia de la masa del LID (con resultado de carcinoma epidermoide) y punción transbronquial aspirativa de las adenopatías mediastínicas (con resultado de muestra no representativa de ganglio linfático). Señale cuál sería la actitud más adecuada:

1. 2. 3. 4. 5.

1. Lobectomía inferior derecha, puesto que se ha descartado razonablemente la naturaleza tumoral de las adenopatías mediastínicas. 2. Realización de test de esfuerzo cardiopulmonar para valorar la operabilidad. 3. Mediastinoscopia, para estudiar las adenopatías mediastínicas. 4. Quimiorradioterapia, ya que se trata de un estadio III-A. 5. RMN cerebral, para estudiar la posible existencia de metástasis cerebrales.

EPOC por déficit de alfa-1-antitripsina. Histiocitosis X. Sarcoidosis pulmonar. Linfangioleiomiomatosis. Fibrosis pulmonar idiopática.

¿Cuál es la primera prueba diagnóstica que hay que realizar en un paciente con sospecha clínica de derrame pleural? 1. 2. 3. 4. 5.

59.

Ecografía torácica. Radiografía posteroanterior y lateral de tórax. TAC torácica. Toracocentesis diagnóstica. TAC torácica helicoidal.

61.

Un paciente de 70 años, fumador, operado de prótesis de cadera, presenta dolor torácico de inicio brusco. En la Rx tórax se objetiva un derrame pleural derecho, con características de exudado hemático. Indique cuál es el diagnóstico más probable: 1. Derrame pleural paraneumónico. 2. Derrame pleural tumoral. 3. Derrame pleural secundario a embolismo pulmonar. 4. Derrame pleural por insuficiencia cardiaca. 5. Quilotórax.

60.

SIMULACRO 5

Acude a su consulta un paciente refiriendo ligera somnolencia diurna y cefalea matutina frecuente. Se trata de un paciente fumador, de 69 años de edad, con un IMC (índice de masa corporal) de 38 kg/m2, que al interrogatorio dirigido refiere tos y expectoración habitual, sin otros síntomas salvo que es roncador habitual. A continuación se detallan las pruebas que le realiza con sus resultados. Rx tórax: normal. Gasometría arterial respirando aire ambiente: pH 7,36, PaCO2 65 mmHg, PaO2 63 mmHg, HCO3 38 mEq/L. Exploración funcional respiratoria: FVC 2430 ml (60%), FEV1 2340 ml (78%), FEV1/FVC 0,91, CPT 4280 ml (65%), VR 1200 ml (85%). Polisomnografía: IAR 4, SatO2 media durante el sueño 80%, SatO2 mínima durante el sueño 69%. Indique cuál, de los siguientes, considera el tratamiento más adecuado en este paciente: 1. Pérdida de peso. 2. Pérdida de peso y ventilación mecánica no invasiva nocturna. 3. Pérdida de peso y CPAP nocturno. 4. Pérdida de peso y tratamiento broncodilatador. 5. Pérdida de peso y estimulantes respiratorios (almitrina o teofilina).

Paciente de 65 años de edad, fumador activo, que consulta por tos y expectoración hemoptoica de 2 meses de evolución. En la Rx tórax se observa una masa pulmonar en el LID. Se le realiza TAC torácica donde se confirma la existencia de una masa sólida en LID, de 6 cm de diámetro, y se observa además una adenopatía paratraqueal derecha de 1.5 cm de diámetro y otra subcarinal de 2 cm de diámetro. Se le realiza una tomografía por emisión de positrones donde se observa captación patológica de fluorodesoxiglucosa a nivel de la masa del LID, y captación indeterminada a nivel de las dos

62.

-11-

Paciente de 53 años de edad, camionero de profesión, fumador y obeso. Acude a su consulta por un cuadro de hipersomnolencia diurna de 2 años de evolución. Su pareja asegura que ronca durante la noche. La gasometría arterial muestra pH 7,38, PaCO2 44 mmHg, PaO2 89 mmHg, HCO3 25 mEq/L. La espi-

booksmedicos.org rometría muestra FVC 4890 ml (89%), FEV1 3400 ml (85%), FEV1/FVC 0,75. Le realiza usted una polisomnografía que muestra los siguientes resultados: IAR 25, SatO2 media durante el sueño 94%, SatO2 mínima durante el sueño 82%. Indique cuál de las siguientes afirmaciones le parece correcta:

4. Tiene una alteración ventilatoria obstructiva con disminución de la difusión. 5. La espirometría es normal, pero la DLCO está disminuída, por lo cual la causa más probable es una alteración vascular.

1. Tiene un síndrome de obesidad-hipoventilación. Hay que recomendar pérdida de peso e iniciar tratamiento con ventilación mecánica no invasiva nocturna. 2. Es un roncador simple. Tan sólo se debe recomendar pérdida de peso. 3. Tiene un síndrome de apnea del sueño. Hay que recomendar pérdida de peso. 4. Tiene un síndrome de apnea del sueño. Hay que recomendar pérdida de peso e iniciar tratamiento con CPAP nocturno. 5. Asocia un síndrome de apnea del sueño con un síndrome de obesidad-hipoventilación, por lo que hay que recomendar pérdida de peso e iniciar tratamiento con CPAP nasal nocturna.

63.

64.

65.

Paciente de 23 años de edad, polínico, que en los últimos 2 meses presenta disnea, sibilancias, tos y opresión precordial, sobre todo al levantarse por la mañana. La exploración física es normal, al igual que la radiografía de tórax. Se le solicita una espirometría forzada cuyos resultados son: FEV1 4140 ml (99%), FVC 5250 ml (104%), FEV1/FVC 79%. ¿Qué prueba debe solicitar a continuación? 1. 2. 3. 4.

Pletismografía. Prueba broncodilatadora. Test de metacolina. Pruebas cutáneas frente a neumoalérgenos habituales, hemograma e IgE total. 5. Test del sudor.

Paciente de 50 años de edad sometido a un trasplante de pulmón bilateral por fibrosis pulmonar idiopática. A los 3 meses de realizado el trasplante el paciente comienza con un cuadro clínico rápidamente progresivo que consiste en tos, disnea, insuficiencia respiratoria, fiebre y aparición de infiltrados pulmonares bilaterales. Señale cuál de las siguientes exploraciones es la más rentable para llegar al diagnóstico: 1. 2. 3. 4. 5.

SIMULACRO 5

66.

Lavado broncoalveolar. Biopsia transbronquial. Hemocultivos. Espirometría. Cultivo de esputo.

Acude a su consulta un paciente de 61 años de edad, de profesión administrativo, no fumador, por un cuadro de disnea de curso progresivo y tos de 8 meses de evolución. No refiere exposición a tóxicos en el ámbito doméstico. En la exploración física se aprecian acropaquias y crepitantes en las bases en la auscultación pulmonar. La radiografía de tórax muestra una afectación intersticial reticular de predominio en bases. ¿Cuál es el diagnóstico más probable? 1. 2. 3. 4. 5.

Una paciente de 35 años está siendo estudiada en consulta de Neumología por tos y disnea de grandes esfuerzos. No es fumadora. Se le solicita un estudio funcional respiratorio que arroja los siguientes resultados: FEV1 2400 ml (62%), FVC 4000 ml (102%), cociente FEV1/FVC 0.60, DLCO 57%. ¿Cuál de las siguientes afirmaciones acerca de este estudio funcional le parece correcta?

67.

1. Tiene una restricción parenquimatosa como muestra el descenso en la difusión de CO. Se debe realizar una pletismografía que lo confirme. 2. Se trata de una alteración ventilatoria restrictiva extraparenquimatosa inspiratoria. 3. La superficie útil de la membrana alveolocapilar es normal.

Sarcoidosis. Fibrosis pulmonar idiopática. Histiocitosis de células de Langerhans. Neumonía intersticial descamativa. Proteinosis alveolar.

Varón de 30 años que acude a su médico de atención primaria por molestias faríngeas y sensación de afonía de 6 meses de evolución. Refería asimismo molestias ocasionales retroesternales. Refería consumo ocasional de magaldrato por pirosis y regurgitación ocasional. Valorado por ORL en la laringoscopia indirecta observa ligero eritema de la región interaritenoidea. ¿Cuál de las siguientes afirmaciones es cierta? 1. Seguramente el paciente presentara datos de esofagitis en una gastroscopia. 2. La causa más probable de la patología que presenta será la infección por H. pylori.

-12-

booksmedicos.org 3. Probablemente mejore con ranitidina, siendo su tratamiento de elección su asociación con antiácidos a demanda. 4. La existencia de otros síntomas respiratorios como el asma o la tos permite descartar la existencia de una enfermedad por reflujo gastroesofágico. 5. El tratamiento con rabeprazol asociado a cinitaprida constituyen un tratamiento eficaz a largo plazo.

68.

1. 2. 3. 4. 5.

71.

Señale, de entre las siguientes, cuál NO es una pauta adecuada para el tratamiento erradicador de la infección por H. pylori: 1. Omeprazol 20 mg/12 horas + Amoxicilina 1 g/12 horas + metronidazol 250 mg/8 horas. 2. Omeprazol 20 mg/12 horas + Amoxicilina 1 g/12 horas + claritromicina 550 mg/12 horas + metronidazol 250 mg/8 horas. 3. Rabeprazol 20 mg/12 horas + Omeprazol 20 mg/12 horas + Amoxicilina 1 g/12 horas. 4. Pantoprazol 40 mg/12 horas + Amoxicilina 1 g/12 horas + levofloxacino 250 mg/12 horas. 5. Pantoprazol 40 mg/12 horas + levofloxacino 250 mg/12 horas + metronidazol 250 mg/8 horas.

69.

70.

Síndrome hipereosinófilo. Estenosis péptica. Achalasia. Esofagitis eosinofílica. Esofagitis candidiásica.

Varón de 36 años que acude a urgencias por importante distensión abdominal, diarrea sanguinolenta de 18 deposiciones al día, fiebre de 40.2º C e importante deterioro del estado general. La analítica practicada aportó los siguientes resultados: Hb 7.9 g/dl, leucocitos 28.000/dl, plaquetas 385.000, fibrinógeno 530, urea 44, creatinina 1.4, GOT 34, GPT 33, bilirrubina 0.6, GGT 30, amilasa 72, LDH 600, sodio 146, potasio 3.2. Los coprocultivos, parásitos y toxina de Clostridium difficile fueron negativos. En la rectoscopia se observaba una mucosa con restos de sangre roja y mucosa friable y con múltiples ulceraciones cubiertas de exudado purulento. Se instauró tratamiento con metilprednisolona a dosis de 60 mg/día, ciprofloxacino 500 mg/12 horas y metronidazol 250 mg/8 horas. A las 48 horas continúa igual, con más dolor. Se realiza una radiografía simple de abdomen donde se comprueba importante dilatación colónica con un transverso de 12 cm y aire libre peritoneal. Señale la opción correcta respecto a lo que haría en esta situación: 1. Es demasiado pronto para observar mejoría, por lo que es prudente esperar. 2. Realizará una colonoscopia completa para valorar la extensión de la enfermedad. 3. Debe plantearse colectomía subtotal urgente. 4. Puede plantearse iniciar tratamiento con ciclosporina i.v. 5. Una alternativa válida es iniciar tratamiento con infliximab i.v.

Mujer de 68 años que acude al ser estudiada por disfagia. En una manometría se observa a nivel cricofaríngeo un tono hipercontráctil con ondas peristálticas en cuerpo esofágico y relajación adecuada del esfínter esofágico inferior. ¿Cuál sería la causa más probable de dicho trastorno? 1. 2. 3. 4. 5.

SIMULACRO 5

Ictus por embolia de la cerebral media. Distrofia muscular de Duchene. Lesión del nervio hipogloso. Esclerodermia. Enfermedad de Pompe.

72.

Varón de 24 años que acude a urgencias por disfagia súbita mientras comía carne y sialorrea. En la endoscopia practicada en urgencias tras extracción del bolo cárnico impactado en esófago medio se observa una mucosa normal. Se toma una biopsia que únicamente demuestra la existencia de 45 eosinófilos por campo en la mucosa. La analítica era la siguiente: Hb 13.6 g/dl, 11.300 leucocitos/ mm3 con neutrófilos 54%, linfocitos 25%, monocitos 13%, eosinófilos 2 %, plaquetas 165.000/mm3, creatinina 1.0, urea 66, GOT 25, GPT 38, GGT 13, bilirrubina 1.1, sodio 147, potasio 4.4. ¿Cuál es el diagnóstico más probable?

Varón de 17 años con síndrome de Down que acude a consulta por diarrea de 4 deposiciones al día líquidas sin productos patológicos. La exploración física era anodina y en la analítica destacaba: Hb 10.9 g/dl, leucocitos 7.000/dl con 55% neutrófilos, plaquetas 225.000, fibrinógeno 210, urea 12, creatinina 1.0, GOT 14, GPT 11, bilirrubina 0.7, GGT 25, amilasa 42, LDH 220, sodio 142, potasio 3.9, proteína C reactiva 4.8 mg/dl, ferritina 5 ng/ml. Los coprocultivos y parásitos fueron negativos. En el estudio inmunológico se apreciaba ANA + a título 1/40, pANCA -, ASCAs -, Acs anti-transglutamidasa tisular + a título 320 UI/ml. ¿Cuál sería la siguiente actitud a realizar en este paciente? 1. Retiraría el gluten de la dieta. 2. Solicitaría cultivo para Cryptosporidium.

-13-

booksmedicos.org 3. Gastroscopia con toma biopsias de 2ª porción duodenal. 4. Test de aliento para H. pylori. 5. Iniciaría tratamiento empírico con rifaximina.

73.

74.

Coledocolitiasis. Fiebre Q. Hepatitis aguda A. Síndrome de Gilbert. Síndrome de Crigler-Najjar.

1. 2. 3. 4. 5.

Varón de 64 años, consumidor de 80 g/día de alcohol, que acude a urgencias por deterioro del nivel de conciencia con respuesta únicamente a estímulos dolorosos. El TC craneal fue normal, sin evidenciarse hematoma subdural. Una ecografía abdominal demostró la existencia de un hígado heterogéneo e irregular con una porta de 18 mm y signos de circulación colateral sin ascitis. La analítica demostró los siguientes parámetros: Hb 11.2, VCM 86.5 fl, HCM 30 pg, ferritina 794 ng/ml, leucocitos 11500 con 72% neutrófilos, plaquetas 95.000, actividad protombina 40%, urea 22 mg/dl, creatinina 0.7 mg/dl, GOT 132, GPT 78, GGT 180, bilirrubina total 3.2 mg/dl, proteínas totales 6 g/dl, albúmina 3.2 g/dl, fosfatasa alcalina 68 U/L, LDH 168, colesterol 236 mg/dl, triglicéridos 86 mg/dl, sodio 142, potasio 3.8. ¿Ante la patología que sospecha, qué grado de Child tendría? 1. 2. 3. 4. 5.

75.

La analítica que presentaba fue la siguiente: Hb 13.2 g/dl, VCM 88.5 fl, HCM 27.3 pg, leucocitos 9500/mm3, plaquetas 205.000/mm3, glucosa 87 mg/ dl, urea 22 mg/dl, creatinina 0.4 mg/dl, GOT 108 U/L, GPT 145 U/L, GGT 111 U/L, bilirrubina total 1.0 mg/dl, proteínas totales 6.2 g/dl, albúmina 4.1 g/dl, fosfatasa alcalina 99 U/L, LDH 122, sodio 139, potasio 3.8. Serología: antiHBs 131 UI/ml, AgHBs -, AntiHBc +, AgHBe -, antiHBe+, DNAVHB -, anti-VHC +, RNA-VHC 854.000 UI/ml, genotipo 1b, AgVHD -, IgG-VHA +. Se inicia tratamiento con interferón pegilado 1.5 microg/kg/ semanal y rivabirina 1000 mg/día. A los 3 meses, la paciente refiere astenia e insomnio. Analíticamente tiene Hb 11.2 g/dl, VCM 87.5 fl, HCM 26.4 pg, leucocitos 7500/mm3, plaquetas 164.000/mm3, GOT 98 U/L, GPT 100 U/L, GGT 122 U/L, bilirrubina total 1.0 mg/dl, fosfatasa alcalina 69 U/L, LDH 182, sodio 137, potasio 4.1. Serología: antiVHC +, RNA-VHC 200.000 UI/ml.¿Cuál sería la actitud más adecuada a continuación?

Varón de 32 años que acude a su consulta de especialista remitido desde atención primaria por leve ictericia desde hace 2 días, fiebre, artromialgias y tos productiva con expectoración ligeramente verdosa. En la exploración física destacaba únicamente sibilancias en ambos campos pulmonares. La Rx tórax era normal. La analítica demostró los siguientes parámetros: Hb 15.2, VCM 86.5 fl, HCM 30 pg, ferritina 94, leucocitos 11500 con 72% neutrófilos, plaquetas 195.000, GOT 32, GPT 23, GGT 32, bilirrubina total 7.8 (directa 0.8), LDH 168, sodio 142, potasio 3.8. ¿Cuál, de entre los siguientes, es el diagnóstico más probable? 1. 2. 3. 4. 5.

SIMULACRO 5

76.

Grado A: 6 puntos. Grado B: 9 puntos. Grado C: 10 puntos. Grado C: 11 puntos. Grado B: 8 puntos.

Continuar tratamiento hasta 48 semanas. Continuar tratamiento hasta 24 semanas. Suspender tratamiento. Añadir entecavir. Retirar la rivabirina por toxicidad excesiva.

Mujer de 62 años con antecedente de diabetes mellitas no insulin-dependiente, hipertensión arterial y EPOC leve acude por astenia y tos. La exploración física reveló ligera disminución de murmullo vesicular en base pulmonar derecha y hepatomegalia de 3 traveses de dedo desde parrilla costal. La Rx de tórax reveló consolidación pulmonar derecha. En la analítica destacaba: Hb 13.5 g/dl, VCM 87.5 fl, HCM 30.3 pg, leucocitos 19500/mm3 con 14300 neutrófilos, plaquetas 215.000/mm3, glucosa 145 mg/dl, urea 32 mg/dl, creatinina 1.1 mg/dl, GOT 214 U/L, GPT 230 U/L, GGT 222 U/L, bilirrubina total 1.0 mg/dl, proteínas totales 6.1 g/ dl, albúmina 4.4 g/dl, fosfatasa alcalina 101 U/L, LDH 127, sodio 142, potasio 4.2. Serología: antiHBs -, AgHBs +, AntiHBc +, AgHBe +, antiHBe-, DNA-VHB 101.000 UI/ml, anti-VHC -, AgVHD +, anti-VHD IgM +, anti-VHD IgG -, IgG VHA +. Señale el diagnóstico más probable: 1. Hepatitis aguda por VHA. 2. Hepatitis crónica B por cepa mutante precore con sobreinfección delta. 3. Hepatitis crónica B por cepa salvaje con sobreinfección delta. 4. Hepatitis aguda B por cepa salvaje con sobreinfección delta. 5. Hepatitis crónica B por cepa mutante precore con coinfección delta.

Mujer de 45 años que en analítica rutinaria se observa hipertransaminasemia de bajo rango. En la exploración física destacaba leve hepatomegalia.

-14-

booksmedicos.org 77.

Varón de 60 años, cirrótico VHC en tratamiento con espironolactona y furosemida a dosis máximas, sin otros antecedentes de interés. Acude a consulta por aumento de perímetro abdominal y dolor en ambos flancos y espalda. En la exploración física se observó abdomen distendido y oleada ascítica. En la analítica destacaba: Hb 11.0 g/ dl, VCM 80.5 fl, leucocitos 10.000/mm3 con 50% neutrófilos, plaquetas 78.000/mm3, glucosa 89 mg/ dl, urea 101 mg/dl, creatinina 2.5 mg/dl, proteínas totales 5.6 g/dl, albúmina 3.3 g/dl. GOT 88 U/L, GPT 101 U/L, GGT 178 U/L, bilirrubina total 1.8 mg/dl, fosfatasa alcalina 108 U/L, LDH 187 U/L, sodio 133, potasio 4.8. ¿Cuál sería la actitud para el control de este paciente? 1. 2. 3. 4. 5.

78.

79.

sencia de captación de contraste en el 65% de la glándula. ¿Cuál sería la actitud a continuación? 1. 2. 3. 4. 5.

80.

TIPS. Paracentesis evacuadora de repetición. Aumentar la dosis de diuréticos. Tratamiento con Peg-IFN y rivabirina. Albúmina i.v. indefinida.

Varón de 56 años que acude a su consulta por hepatomegalia y la siguiente analítica: Hb 13.2 g/dl, leucocitos 6500/mm3, plaquetas 175.000/mm3, glucosa 111 mg/dl, urea 50 mg/dl, creatinina 0.7 mg/dl, GOT 178 U/L, GPT 97 U/L, GGT 100 U/L, bilirrubina total 1.0 mg/dl, fosfatasa alcalina 111 U/L, alfafetoproteína 1080 ng/ml. AntiHBs -, AgHBs +, AntiHBc +, AgHBe -, antiHBe+, DNA-VHB +, anti-VHC -, AgVHD -. Se realiza una ecografía abdominal en la que se observa un hígado heterogéneo e irregular con una lesión nodular de 4.5 cm. En un TC se observan los mismos hallazgos, siendo una lesión con realce en fase arterial y lavado precoz en fase venosa. ¿Cuál sería el diagnóstico más probable? 1. 2. 3. 4. 5.

SIMULACRO 5

Varón de 56 años, bebedor y fumador, que acude a urgencias por ictericia de 1 semana evolución. La exploración física fue anodina. La radiografía simple de abdomen mostraba un patrón gaseoso inespecífico. La analítica realizada fue la siguiente: Hb 14.2 g/dl, leucocitos 7.300/mm3, plaquetas 199.000/ mm3, glucosa 170 mg/dl, urea 55 mg/dl, creatinina 0.8 mg/dl, albúmina 5.0 g/dl, GOT 97 U/L, GPT 140 U/L, GGT 665 U/L, bilirrubina total 4.7 mg/dl, fosfatasa alcalina 380 U/L, LDH 300 U/L, amilasa 18 U/ml, sodio 140, potasio 4. La ecografía abdominal fue normal. Una colangio-RMN demostró dilatación moderada de vía biliar intra y extrahepática con wirsung también dilatado con estenosis única en cabeza. ¿Cuál sería diagnóstico más probable del paciente? 1. 2. 3. 4. 5.

81.

Metástasis de cáncer de colon. Cirrosis hepática VHC. Hepatocarcinoma. Angioma hepático. Coinfección VHB-VHD.

Mujer de 66 años que acude a urgencias por dolor en epigastrio y náuseas y vómitos alimentario-biliosos. La exploración física es de dolor epigástrico a la palpación con defensa y ruidos hidroaéreos negativos. La analítica fue: Hb 13.2 g/dl, leucocitos 19.500/mm3, plaquetas 195.000/mm3, glucosa 111 mg/dl, urea 40 mg/dl, creatinina 0.7 mg/dl, GOT 478 U/L, GPT 190 U/L, GGT 200 U/L, bilirrubina total 1.4 mg/dl, fosfatasa alcalina 311 U/L, amilasa 1080 U/L. Se realiza un TC abdominal con contraste i.v. en el que se observa un páncreas edematoso con colecciones peripancreáticas y au-

Necrosectomía profiláctica. Imipenem i.v. Eritromicina oral. Somatostatina i.v. CPRE.

Pancreatitis aguda. Pancreatitis crónica cefálica. Coledocolitiasis. Carcinoma páncreas. Pancreatitis del surco.

Varón de 67 años, hipertenso, diabético y obeso que acude al servicio de urgencias por diarrea de 5 días de evolución que en las últimas 8 horas son claramente sanguinolentas y mareo. La exploración física revelaba sequedad de piel y mucosas y distensión abdominal leve con dolor difuso. La radiografía simple de abdomen descartaba perforación. Se realizó una colonoscopia en la que se objetiva en unión recto-sigma una mucosa eritematosa y friable con hematomas submucosos. ¿Cuál sería la actitud MENOS adecuada en este momento? 1. Colectomía urgente. 2. Reposición hidroelectrolítica. 3. Tratamiento conservador con antibiótico de amplio espectro. 4. Analgesia. 5. Coprocultivos.

82.

-15-

Varón de 25 años que acude su consulta por rectorragia y astenia. La exploración física era ano-

booksmedicos.org dina salvo hiperpigmentación en el borde interno de labio inferior. La analítica realizada fue la siguiente: Hb 11.2 g/dl, plaquetas 180.000/mm3, leucocitos 73.000 con 4500 neutrófilos, glucosa 94 mg/ dl, urea 29 mg/dl, creatinina 0.8 mg/dl, bilirrubina total 0.4 mg/dl, GOT 43 U/l, GPT 38 U/l, GGT 23 U/l, fosfatasa alcalina 59 U/l, amilasa 34 U/l, LDH 190 U/l, PCR 0.3 mg/l, sodio 142, potasio 4.2. Los coprocultivos y parásitos fueron negativos. La colonoscopia demostró hasta 15 pólipos pediculados en colon izquierdo e íleon. ¿Cuál de entre las siguientes afirmaciones le parece correcta sobre el diagnóstico que sospecha?

con cuatro fármacos, en línea con las últimas recomendaciones. Tanto la función hepática como el hemograma en la evaluación inicial fueron normales. Al cabo de 3 semanas de tratamiento le consulta refiriendo que ve peor las indicaciones de tráfico y que, en ocasiones, ha llegado a “confundir los colores del semáforo”. ¿Cuál de los siguientes fármacos considera que puede estar implicado en el cuadro? 1. 2. 3. 4. 5.

1. Debe realizarse screening de tumores gonadales. 2. Los pólipos aparecen típicamente en colon izquierdo respetando el intestino delgado. 3. Se trata de una poliposis adenomatosa atenuada y, por tanto, el tratamiento debe ser endoscópico. 4. No malignizan nunca esos pólipos. 5. Es frecuente la presencia de hipertrofia del epitelio pigmentario de la retina.

83.

85.

Paciente de 55 años, diagnosticado de artritis reumatoide desde hace 5 años, que sigue tratamiento crónico con prednisona (10 mg diarios) y metotrexate. Consulta por un cuadro de 10 días de evolución consistente en cefalea progresiva, tendencia a la somnolencia y fiebre. A la exploración física presenta un GCS de 13 puntos, nistagmo horizontal, paresia de los pares craneales VI, IX y XII derechos y ataxia. Tras descartar mediante un TC craneal la existencia de signos de hipertensión intracraneal se practica una punción lumbar. Señale cuál de las siguientes combinaciones de hallazgos en el LCR y agente etiológico considera MÁS PROBABLE:

Isoniacida. Rifampicina. Pirazinamida. Estreptomicina. Etambutol.

Mujer de 45 años, natural de Cochabamba (Bolivia) y residente en nuestro país desde hace 5 años. Tras consultar por un cuadro de disnea de esfuerzo progresivo y edemas en miembros inferiores, se practica una radiografía de tórax (que evidencia una cardiomegalia notable con signos de redistribución vascular en campos pulmonares superiores) y un ECG (con bloqueo completo de rama derecha y frecuentes extrasístoles ventriculares). La paciente niega el consumo de tóxicos. Tanto sus cifras tensionales habituales como los niveles de glucemia, colesterol y ácido úrico son normales. Reinterrogada detalladamente refiere que parte de su infancia la pasó en un medio rural, en viviendas de adobe, y que un hermano mayor falleció de forma súbita a los 18 años “mientras hacía ejercicio”. Respecto a la enfermedad que muy probablemente presente la paciente, señale la opción CORRECTA: 1. Su distribución geográfica incluye amplias áreas de Centro y Sudamérica, África Subsahariana y el Sudeste Asiático. 2. Se transmite por la picadura de la hembra del mosquito Aedes albopictus. 3. La forma aguda de la enfermedad asocia en más del 90% de los casos cuadros de miocarditis y encefalitis aguda frecuentemente mortales. 4. El tratamiento con benznidazol resulta especialmente mal tolerado en niños. 5. El diagnóstico durante la fase crónica se basa en técnicas serológicas.

1. Pleocitosis polimorfonuclear, discreta hipoglucorraquia e hiperproteinorraquia. Streptococcus pneumoniae. 2. Pleocitosis mononuclear, glucorraquia y proteinorraquia normales. Virus herpes simple tipo 2. 3. Celularidad normal, hipoglucorraquia marcada e hiperproteinorraquia. Pseudomomas aeruginosa. 4. Pleocitosis linfocitaria, hipoglucorraquia e hiperproteinorraquia. Aspergillus fumigatus. 5. Pleocitosis linfocitaria, hipoglucorraquia marcada e hiperproteinorraquia. Listeria monocytogenes. 86. 84.

SIMULACRO 5

Un paciente de 37 años, conductor profesional y sometido a tratamiento esteroideo por una colitis ulcerosa, ha sido diagnosticado hace un mes de enfermedad tuberculosa en forma de reactivación pulmonar con una lesión cavitada en lóbulo superior izquierdo. Por este motivo inició tratamiento

¿Cuál de los siguientes antibióticos está formalmente contraindicado en pacientes con miastenia gravis? 1. Cloranfenicol. 2. Amikacina. 3. Vancomicina.

-16-

booksmedicos.org 4. Cefoxitina. 5. Penicilina G benzatina.

87.

88.

1. El agente causal se transmite al ser humano por contacto directo con el huésped o mediante la inhalación de esporas. 2. No existe transmisión de persona a persona. 3. Hasta la tercera parte de los casos se asocian a la elevación de las transaminasas. 4. El diagnóstico de la fase aguda se establece a través de la positivización de los anticuerpos dirigidos contra los antígenos de fase I. 5. El tratamiento de elección es la doxiciclina.

90.

Abacavir. Emtricitabina. Darunavir. Lamivudina. Tenofovir.

Un varón de 18 años, sin ningún antecedente de interés, acude al Servicio de Urgencias por presentar desde hace 5 días febrícula sostenida, tos escasamente productiva, cefalea, otalgia y artromialgias. En la radiografía simple de tórax se observa un infiltrado pulmonar intersticial bilateral. Analíticamente destacan los siguientes datos: 10.000 leucocitos/mm3, 9,4 g/dL de hemoglobina, reticulocitos del 2%, haptoglobina disminuida y elevación de los niveles de LDH (950 UI/L). ¿Cuál de los siguientes considera que es el agente causal más probable del cuadro que presenta el paciente? 1. 2. 3. 4. 5.

89.

ductiva, artromialgias y cefalea. A la auscultación pulmonar presentaba crepitantes finos en ambas bases. La radiografía de tórax mostró un infiltrado intersticial bilateral micronodular. Respecto a la enfermedad que probablemente presente el paciente, señale la opción INCORRECTA:

Una mujer de 28 años, gestante de 5 semanas, ha sido recientemente diagnosticada de infección por VIH-1 durante el screening rutinario realizado por su ginecólogo. La paciente niega sintomatología alguna sugerente de infección oportunista previa. Su recuento de linfocitos T-CD4+ es de 220 células/ mcL y la carga viral de VIH-1 de 90.000 copias/mL. El resto del estudio serológico incluye VHA (IgG positivo), VHB (anti-HBs positivo, IgG anti-HBc positivo, anti-HBe positivo, Ag HBs negativo), VHC (IgG y RNA positivo) y Toxoplasma (IgG positivo). La ecografía obstétrica del primer trimestre es normal. Analíticamente presenta discreta anemia (Hb 10 g/dL), hipercolesterolemia (colesterol total 220 mg/dL, cHDL 50 mg/dL, cLDL 190 mg/dL) e hiperuricemia. Usted solicita, entre otras, las siguientes pruebas complementarias: PPD (positivo con 6 mm), tropismo viral (R5), determinación del haplotipo HLB B*5701 (positivo), y genotipo del VHC (1b). Decidido a iniciar tratamiento antirretroviral, señale cuál de los siguientes fármacos considera CONTRAINDICADO en esta paciente: 1. 2. 3. 4. 5.

SIMULACRO 5

Mujer de 34 años, fumadora de 2 paquetes diarios y sin otros antecedentes de interés. Consulta por un cuadro de 48 horas consistente en fiebre elevada (39 ºC) con escalofríos, tos con abundante expectoración purulenta, dolor de naturaleza pleurítica en el hemitórax izquierdo, y malestar general. A la exploración física presenta TA 130/65 mmHg, FC 100 lpm, FR 18 rpm, y destaca un herpes labial y la auscultación de crepitantes y soplo tubárico en la base pulmonar izquierda. Analíticamente resultan reseñables los siguientes datos: 15.000 leucocitos/mm3 (95% de neutrófilos), 12,4 g/dL de hemoglobina, 195.000 plaquetas/mm3, pH 7,345, pO2 basal de 95 mmHg. En la radiografía simple de tórax se observa un infiltrado alveolar con broncograma aéreo en el lóbulo inferior derecho, sin derrame pleural asociado. ¿Cuál de las siguientes considera que debe ser la actitud terapéutica más apropiada? 1. Ingreso hospitalario y tratamiento con meropenem. 2. Alta a domicilio y tratamiento con azitromicina. 3. Ingreso hospitalario y tratamiento con ceftriaxona y claritromicina. 4. Alta a domicilio y tratamiento con doxiciclina. 5. Alta a domicilio y tratamiento con moxifloxacino.

Streptococcus pneumoniae. Coxiella burnetii. Mycoplasma pneumoniae. Chlamydophila pneumoniae. Legionella pneumophila.

91.

Varón de 67 años, ganadero de profesión, consulta por un cuadro de febrícula, tos escasamente pro-

-17-

Mujer de 44 años, usuaria activa de drogas por vía parenteral, consulta por fiebre con escalofríos en las últimas 72 horas. A la exploración física presenta buen estado general, TA 110/65 mmHg, FC 89 lpm, FR 12 rpm, saturación basal de O2 96%, estigmas de venopunción en ambos miembros superiores, y ausencia de soplos a la auscultación cardíaca. La radiografía simple de tórax es normal. A las 25 horas de instaurar tratamiento anti-

booksmedicos.org

92.

biótico empírico le informan desde el Servicio de Microbiología del aislamiento de Staphylococcus aureus sensible a la oxacilina (SAOS) en todos los hemocultivos extraídos inicialmente. Tras confirmar mediante un ecocardiograma transtorácico su sospecha diagnóstica, ¿cuál de las siguientes considera que debe ser la actitud terapéutica más apropiada?

de una adenopatía móvil en la axila derecha. Tras una anamnesis detallada el paciente refiere que mantiene relaciones sexuales habitualmente no protegidas, que convive con un perro y dos gatos, y que dos días antes se pinchó con una espina de atún en el dedo índice de la mano derecha. Señale cuál es el agente más probable de este cuadro, y qué tratamiento considera indicado:

1. Cloxacilina y gentaminicina durante 6 semanas, sin reparación quirúrgica. 2. Cloxacilina y gentamicina durante 4 semanas, seguidas de recambio valvular. 3. Vancomicina y gentamicina durante 4 semanas, sin reparación quirúrgica. 4. Reparación quirúrgica inmediata, seguida de cloxacilina y gentamicina, al menos durante 4 semanas. 5. Cloxacilina y gentamicina durante 2 semanas, sin necesidad de reparación quirúrgica.

1. 2. 3. 4. 5.

94.

Varón de 83 años, ingresado en la planta de Medicina Interna desde hace dos semanas como consecuencia de una exacerbación de perfil infeccioso de EPOC. Por este motivo ha recibido varios ciclos de tratamiento antibiótico con ceftriaxona (1 g/24 horas) y levofloxacino (500 mg/24 horas). Desde hace 24 horas ha comenzado con varias deposiciones líquidas, acompañadas de moco y dolor abdominal tipo cólico, sin fiebre, náuseas ni otra sintomatología sistémica. La exploración abdominal no muestra signos de irritación peritoneal y la radiografía simple de abdomen es normal. Respecto al cuadro que muy probablemente presenta el paciente, señale la opción CORRECTA:

Francisella tularensis. Estreptomicina. Sporothrix schenckii. Itraconazol. Bacillus anthracis. Penicilina G. Staphylococcus aureus. Cloxacilina. Erysipelothrix rhusiopathiae. Penicilina G.

El paciente A presenta una serología de sífilis con los siguientes resultados: FTA positivo; RPR positivo 1/2. El paciente B presenta una serología con los siguientes resultados: TPHA negativo; VDRL positivo 1/64. Ninguno de los dos pacientes presenta clínica sugestiva de ninguna de las fases de la sífilis. Señale la opción que considera CORRECTA: 1. Paciente A: sífilis tratada y curada; Paciente B: falso negativo de la prueba treponémica. 2. Paciente A: sífilis tratada y curada; Paciente B: falso positivo de la prueba reagínica. 3. Paciente A: sífilis latente; Paciente B: sífilis tratada y curada. 4. Paciente A: ambas pruebas falsamente positivas; Paciente B: sífilis primaria. 5. Paciente A: latencia tardía; Paciente B: latencia precoz.

1. Su agente etiológico es un bacilo grampositivo aerobio no esporulado. 2. El principal efecto patógeno está mediado por la enterotoxina A. 3. No existe el estado de portador crónico asintomático para este microorganismo. 4. El coprocultivo constituye la técnica diagnóstica de elección. 5. Todos los antibióticos, incluido el metronidazol y la vancomicina, pueden contribuir en mayor o menor medida al desarrollo de esta complicación.

93.

SIMULACRO 5

95.

Varón de 53 años, alcohólico, diagnosticado de tuberculosis pulmonar con cultivo de esputo positivo para M. tuberculosis. Se realizó estudio de contactos con los siguientes resultados: esposa de 47 años con prueba de Mantoux negativa e hijo de 12 años con Mantoux negativa. ¿Qué actitud le parece la más adecuada? 1. Repetir la prueba de Mantoux a las 8 semanas a los dos convivientes. 2. Repetir la prueba de Mantoux a la esposa a las 8 semanas e iniciar inmediatamente tratamiento con isoniazida en el hijo. 3. En ambos casos queda descartada la infección tuberculosa y no son necesarias más pruebas. 4. Iniciar en ambos casos tratamiento con isoniazida en espera del resultado de la prueba de Mantoux al cabo de 8 semanas. 5. Todas la anteriores son falsas.

Un varón de 30 años, fumador habitual de medio paquete al día, pescadero de profesión y cazador de palomas torcaces en sus ratos libres, consulta por presentar una mácula eritematosa, caliente y dolorosa en el brazo derecho. A la exploración física destaca además la existencia de febrícula, de lesiones vesiculosas sobre el área de eritema, y

-18-

booksmedicos.org 96.

Varón de 75 años con antecedentes de HTA, DM, 2 AITs previos, que desde hace 2 años comienza con clínica de alteración de la marcha progresiva, compuesta sobre todo por desestructuración de la misma, con pasos cortos y problemas para la realización de giros. En los últimos meses el paciente presenta además problemas para la retención de datos, así como para recordar hechos recientes y el nombre de los familiares. No se acompaña de movimientos anormales. A la exploración presenta pérdida de fuerza 4+/5 en extremidades inferiores de forma simétrica, con reflejos hipoactivos. No problemas ni sensitivos. No dismetrías ni disdiadococinesias, Romberg negativo. Según refiere la familia ha sido necesario la colocación de pañales sin saber precisar bien desde cuando. El cuadro que presenta el paciente es compatible con:

ciente presenta GCS 15, sin focalidad neurológica, aunque persiste algo de cefalea y sensación de mareo. Se he realizado Rx craneal donde se observa fractura lineal temporal derecha. La actitud más correcta en el momento actual sería: 1. Ingreso en UCI para observación. 2. Realización de TC craneal para descartar lesiones intracraneales y, si está neurológicamente estable, remitir a domicilio tras observación de unas 6 horas. 3. Realización de TC craneal para descartar lesiones intracraneales e ingreso de 1 semana para vigilancia. 4. Alta domiciliaria pues el paciente está asintomático. 5. Observación 24 horas.

1. Deterioro del estado general dependiente de la edad. 2. Hidrocefalia normotensiva. 3. Enfermedad de Parkinson. 4. Demencia vascular. 5. Enfermedad de Alzheimer.

97.

99.

Varón de 54 años, HTA, fumador y con FA, que debuta de forma brusca con problemas para hablar y pérdida de fuerza en brazo derecho. A la exploración el paciente se encuentra consciente, alerta, orientado. Afebril. Entiende de forma coherente las órdenes que se le solicitan, si bien tiene un problema franco para la nominación, no consiguiendo nombrar adecuadamente 3 de cada 4 objetos. La senibilidad es normal, igual que la fuerza en hemicuerpo izquierdo. En el hemicuerpo derecho apenas levanta contra gravedad el brazo derecho, así como presenta desviación de la comisura bucal, sin afectación de la pierna. A la llegada a urgencias se le realiza una TC craneal que es informada como “... lesión hipodensa con captación de contraste en anillo, con importante edema asociado...”. Dentro del diagnóstico diferencial de la lesión que presenta este paciente no consideraría: 1. 2. 3. 4. 5.

Valora usted a un paciente de 43 años. Entre sus antecedentes personales destacan dos ingresos en el último año por neumonía. En el último desarrolló odinofagia progresiva siendo diagnosticado de candidiasis esofágica y posteriormente de infección por VIH. Consulta por dificultad para la marcha de semanas de evolución. A la exploración se objetiva paraparesia de predominio en flexores de cadera y rodilla, más marcada en lado derecho. Los reflejos rotulianos se encuentran exaltados, mostrando clonus aquíleo derecho. El cutáneo plantar es extensor bilateralmente. El examen de la sensibilidad muestra alteración propioceptiva en ambos miembros inferiores. Respecto al cuadro que presenta el paciente, señale la opción correcta: 1. El paciente presenta una polineuropatía asociada al VIH. Solicitaría un electromiograma. 2. Probablemente presenta una compresión hemimedular extrínseca de origen neoplásico. Solicitaría RM lumbar. 3. El déficit que muestra se corresponde con la afectación centromedular secundaria a una mielopatía por VIH. Solicitaría RM medular. 4. El paciente muestra un cuadro compatible con lesión de las columnas posterolaterales. Solicitaría RM medular. 5. Probablemente se trate de una mielorradiculitis infecciosa. Solicitaría RM medular y punción lumbar.

Glioma de bajo grado. Glioblastoma multiforme. Metástasis cerebral. Absceso. Toxoplasmosis. 100.

98.

SIMULACRO 5

Se presenta en la urgencia un varón de 32 años que dice que ha sufrido un impacto en región temporal derecha, con pérdida de conocimiento de 20 segundos de duración, con posterior recuperación inmediata y sin secuelas. A la exploración el pa-

-19-

En el examen de un paciente inconsciente se objetiva ausencia de respuesta a estímulos dolorosos. Las pupilas se muestran medias y arreactivas. Los reflejos oculocefálicos verticales están abolidos, conservándose los horizontales. El reflejo corneal está presente de forma bilateral, siendo imposible

booksmedicos.org evocarse los espinociliares. El cutáneo plantar es extensor de forma bilateral. Respecto al coma de este paciente señale la verdadera:

repetidas. Los reflejos se encuentran hipoactivos. Respecto al cuadro que sospecha, es falso: 1. Se pueden encontrar anticuerpos anti-canal de calcio en el 95% de los pacientes. 2. Debido a la asociación con tumores subyacentes, en especial de pulmón, solicitaría una prueba de imagen torácica. 3. La estimulación repetida a altas frecuencias durante el EMG produce respuesta decremental. 4. El potencial de acción ante un estímulo único es de escasa amplitud. 5. Es frecuente encontrar alteraciones pupilares.

1. Probablemente se trate de un coma tóxico-metabólico por lo que solicitaría gasometría, bioquímica completa y análisis de tóxicos. Comenzaría a administrar naloxona y flumacenilo de inmediato. 2. La exploración es indicativa de lesión estructural, situando el nivel de la lesión en la región bulbar. Solicitaría un TC craneal urgente. 3. La exploración es indicativa de lesión estructural, situando el nivel de la lesión en la región pontina. Solicitaría un TC craneal urgente. 4. La exploración es indicativa de lesión estructural, situando el nivel de la lesión en la región hemisférica derecha. Solicitaría un TC craneal urgente. 5. La exploración es indicativa de lesión estructural, situando el nivel de la lesión en la región mesencefálica. Solicitaría un TC craneal urgente.

101.

103.

Un paciente de 63 años, sin antecedentes de interés, consulta por diplopia de dos semanas de evolución. Refiere que el trastorno ha ido empeorando en los últimos días, asociando además torpeza de la marcha. La exploración pone de manifiesto parálisis parcial de ambos terceros pares, con pupilas hiporreactivas. La marcha es atáxica y en el examen de los reflejos llama la atención la abolición de ambos aquíleos y rotulianos, siendo el resto hipoactivos. El paciente refiere que hace 3 semanas sufrió una gastroenteritis infecciosa que le duró una semana. Respecto al cuadro que presenta señale la falsa:

Un niño de 3 años es traído a consulta por trastorno de la marcha. El niño completó correctamente los hitos del desarrollo, adquiriendo la marcha a los 13 meses. Desde hace 3 meses el niño muestra una marcha torpe, con claudicación de la pelvis en el apoyo monopodal, y tiene grandes problemas para levantarse del suelo. A la exploración se evidencia hipertrofia de ambos tríceps surales, y debilidad marcada de la cintura pelviana. La CPK es de 488. Un primo por vía materna tuvo una enfermedad muscular y falleció de neumonía a los 17 años. Señale el diagnóstico más probable: 1. 2. 3. 4. 5.

1. El examen del líquido cefalorraquídeo puede mostrar hiperproteinorraquia sin pleocitosis asociada. 2. Se pueden identificar en suero anticuerpos anti GQ1b. 3. El pronóstico es peor que las formas clásicas, con escasa recuperación. 4. Supone un 5% de los casos de síndrome de Guillain-Barré. 5. Es frecuente la presencia de parálisis facial, incluida la afectación bilateral.

102.

SIMULACRO 5

104.

Un varón de 75 años hipertenso y fumador activo de 2 paquetes/año consulta por pérdida de fuerza en miembros inferiores. El paciente refiere dificultad para levantarse de asientos bajos y para subir escaleras. Ocasionalmente ha presentado diplopia variable y visión borrosa. A la exploración se observa cierta pérdida de fuerza de predominio en cintura pélvica, que mejora tras unas pocas contracciones

Distrofia muscular de Duchenne. Distrofia muscular de Becker. Síndrome de MELAS. Distrofia muscular de Steinert. Miopatía nemalínica.

Consulta a la urgencia un varón de 42 años por pérdida progresiva de fuerza en el brazo derecho y alteración del lenguaje. A la exploración el paciente presenta afasia motora sin alteración de otras funciones superiores ni clínica sensitiva ni cerebelosa. La fuerza es normal, salvo por presentar clara claudicación con las maniobras antigravitatorias en MSD. Se realiza TC craneal que muestra una lesión hipodensa fronto-temporal izquierda, sin calcio ni quistes en su interior, que no genera efecto masa y que no realza tras la administración de contraste. En relación a la patología que se sospecha en este paciente es falso que: 1. La lesión más probable es un glioma de alto grado. 2. La opción de vigilancia estrecha con pruebas de imagen seriadas es factible, pues se encuentra localizada en un área elocuente. 3. Si se decide tratamiento agresivo, el marcador pronóstico más claramente establecido es la resección completa de la lesión.

-20-

booksmedicos.org 4. Es útil el inicio de tratamiento antiepiléptico. 5. La RM cerebral nos dará más datos acerca de la posible etiología de la lesión.

105.

107.

Mujer de 68 años de edad, diabética, hipertensa, dislipémica y obesa, que consulta por dolor en miembros inferiores y dificultad progresiva para la marcha desde hace 1 año El dolor comienza en la región glútea bilateral e irradia por cara posterior de ambos miembros inferiores. Aparece cuando lleva caminando unos 500 metros, mejorando a los pocos minutos de haberse parado a descansar y sentarse flexionando el tronco hacia delante. Cuál de las siguientes opciones es incorrecta:

Mujer de 46 años, fumadora e hipertensa, que consulta por cefalea de aparición brusca hace 2 horas. La cefalea comenzó mientras estaba moviendo unos muebles de su casa, predomina en la región occipital y es opresiva y de gran intensidad, asociando nauseas y fotofobia. Refiere cefaleas de menor intensidad durante la última semana a las que no ha dado importancia. En la exploración la paciente presenta buen nivel de conciencia, no tiene focalidad neurológica, los signos meníngeos son positivos y presenta rigidez de nuca. Respecto a la patología que presenta la paciente, señale la opción incorrecta: 1. La TC craneal mostrará sangre en las cisternas basales en > 95% de los casos en las primeras 48 horas. 2. Si la radiología no muestra sangrado pero el grado de sospecha sigue siendo alto está indicado realizar una punción lumbar. 3. Debe realizarse una angiografía de 4 vasos lo antes posible para filiar el origen del sangrado. 4. En caso de que la angiografía sea negativa el paciente puede ser dado de alta a su domicilio. 5. La presencia de xantocromía en el LCR puede detectarse a partir de las 4-6 horas tras el sangrado y permanecer hasta 3 semanas tras el mismo.

1. En la fisiopatología de este cuadro clínico se produce un compromiso vascular de las raíces de la cola de caballo. 2. La RM lumbar probablemente mostrará un canal vertebral estrecho con morfología triangular. 3. El EMG puede ser normal. 4. Hay que establecer el diagnóstico diferencial con la claudicación de origen vascular. 5. El tratamiento quirúrgico de elección consiste en la realización de una laminectomía cervical.

106.

SIMULACRO 5

108.

Varón de 56 años, diagnosticado de enfermedad de Parkinson desde hace 15 años, que ha empeorado en los últimos años a pesar de haber aumentado la dosis de su medicación habitual y haber combinado varios de ellos. En la exploración se aprecia predominio de la rigidez y la bradicinesia. Debido a que se encuentra claramente discapacitado para realizar su vida normal se le propone cirugía. Señale cuál de las siguientes afirmaciones no es correcta respecto a las técnicas disponibles para esta enfermedad: 1. Actualmente el tratamiento de elección es la implantación de electrodos de estimulación profunda en ambos núcleos subtalámicos. 2. En caso de que predomine el temblor, puede optarse por el núcleo ventral intermedio (VIM) como diana de los electrodos. 3. Las principales complicaciones de la cirugía son la hemorragia, la infección y las alteraciones neurológicas secundarias a la propagación del estímulo a zonas adyacentes. 4. La medicación de los pacientes debe retirarse por completo una vez que los electrodos comiencen a estimular ya que interfiere en la actividad de los mismos. 5. La batería de los electrodos debe recambiarse cuando esté a punto de agotasre.

Paciente varón de 32 años de edad, con AP de HTA de 3 años de evolución y fumador de 5 cig/día, que acude urgencias por haber presentado un único episodio de hematuria macroscópica. Interrogado ha estado tomando durante los 4 días previos al ingreso paracetamol y n-acetilcisteína por infección de VRS de probable origen viral. EF destaca TA 160/78 mmHg, FC 85 lpm, Tª 37º C, resto sin hallazgos. En analítica de urgencias presenta Hb 13 g/dl, Leucocitos: 11000 (Linfocitos 45%), Plaquetas 272000, Cr 1,3 mg/dl, Urea 44 mg/dl, Na 137 meq/l, K 4,2 meq/l, Alb 4 g/dl. En este caso la actitud más adecuada sería: 1. Realizar estudio inmunológico, administrar bolos de metilprednisolona y biopsia renal urgente. 2. Realizar estudio inmunológico, administrar bolos de metilprednisolona + ciclofosfamida oral. 3. Derivar a MAP. Se sospecha GN mesangial de Ig A y en ningún caso va a precisar tratamiento inmunosupresor ni biopsia. 4. Realizar estudio inmunológico e ingreso en planta con tratamiento sintomático, evitando la administración de corticoides, ya que con toda probabilidad se trata de una GN postinfecciosa. 5. Realizar estudio inmunológico, cuantificar proteinuria y valorar necesidad de biopsia renal programada.

-21-

booksmedicos.org 109.

Paciente de 78 años de edad sin AP de interés que acude a su MAP por dolores óseos generalizados desde hace meses. Interrogada refiere astenia, disminución del apetito y posible pérdida de peso no cuantificada. Se realiza analítica de sangre donde destaca Hb 7,9 g/dl, Hcto 23,5%, Leucocitos 6500 (FN), Plaquetas 175000, Glu 96 mg/dl, Cr 2,1 mg/ dl, Urea 84 mg/dl, Na 137 meq/l, K3,7 meq/l, Ca 10,9 mg/dl, P 4,6 mg/dl, Alb 3,4 g/dl. La sospecha diagnóstica inicial será: 1. 2. 3. 4. 5.

Tª 36,9º C. En analítica de sangre destaca Hb 7,4 g/ dl, Hcto 23%, Plaquetas 245000, Leucocitos 10500 (80% Neutrófilos) Glu 88 mg/dl, Cr 6 mg/dl, Urea 244 mg/dl, Na 136 meq/l, K 5,5 meq/l. Inmunoglobulinas en rango, C3 98 mg/dl, C4 30 mg/dl, c-ANCA negativo, p-ANCA 1:160, anticuerpos AMBG negativos. En orina 5-10 hematíes por campo y proteinuria +++. El diagnóstico más probable será: 1. 2. 3. 4. 5.

Amiloidosis AL. Sarcoidosis. Poliangeítis microscópica. Mieloma múltiple. Purpura trombocitopénica trombótica. 113.

110.

Paciente mujer de 22 años de edad que ingresa en UCI tras intento autolítico. En la analítica de urgencias destaca: Ph 7,2, Bic 10 meq/l, CO2 25 mmHg, Glu 92 mg/dl, Urea 42 mg/dl, Cr 1,2 mg/ dl, Na 142 meq/l, Cl 101 meq/l. Osm medida en plasma: 335 mosm/l. Lo más probable es que se trate de una intoxicación por: 1. 2. 3. 4. 5.

111.

112.

AAS. Benzodiacepinas. Paracetamol. Metanol. Barbitúricos.

Panarteritis microscópica. Crioglobulinemia mixta esencial. Enfermedad de Good-Pasture. Sdr. Churg-Strauss. Granulomatosis de Wegener.

Paciente mujer de 72 años sin AP de interés que acude a urgencias por cuadro de hemiparesia izquierda que desaparece a las 2 horas del ingreso. Durante el interrogatorio presenta cuadro de disartria de pocos minutos de duración, refiere MEG de varios días de evolución y aparición de petequias en MMSS y MMII. En la analítica de urgencias se objetiva: Hb 7,4 g/dl, Hcto 22%, Plaquetas 18.000, Leucocitos 9000 (FN), Glu 82 mg/ dl, Urea 80 mg/dl, Cr 1,8 mg/dl, iones en rango. En el frotis de sangre abundantes esquistocitos. Respecto al proceso que presenta la paciente señale la respuesta correcta: 1. El tratamiento con plasmaféresis debe ser iniciado lo más precozmente posible. 2. El tratamiento antiagregante ha demostrado gran eficacia, tanto en monoterapia como combinado con plasmaféresis. 3. Es fundamental iniciar tratamiento con corticoides sistémicos y sólo si la paciente presenta mala evolución asociaremos plasmaféresis. 4. El tratamiento debe ser combinado asociando corticoides, antiagregantes, plasmaféresis y trasfusión de plaquetas. 5. La administración de corticoides en estos pacientes está contraindicada.

Paciente varón de 44 años de edad con AP de HTA y ERC en programa de hemodiálisis crónica. En la analítica mensual para control de la anemia destaca: Hb 14 g/dl, Hcto 34%, Plaquetas 215000, Leucocitos 8800 (FN). Estudio del hierro: IST 28%, Ferritina 110 mg/dl. No ha precisado administración de eritropoyetina ni hierro i.v. desde que inició programa de hemodiálisis. La etiología más probable de su insuficiencia renal será: 1. 2. 3. 4. 5.

SIMULACRO 5

Nefropatía diabética. Poliquistosis del adulto. Nefroangioesclerosis. Glomerulonefritis primaria. Enfermedad quística medular.

114.

Paciente varón de 86 años de edad. Traído por su familia al hospital por cuadro clínico de astenia intensa y tos con expectoración hemoptóica de 48 horas de evolución. No precisan ritmo de diuresis ya que orina en pañal. Presenta TA 148/92 mmHg, FC 98 lpm,

Paciente varón de 76 años de edad diagnosticado de mieloma múltiple con proteinuria de Bence-Jones, presenta la siguiente analítica: pH 7,30, PCO2 34 mmHg, HCO3 18 , Glu 88 mg/dl, Urea 75 mg/ dl, Cr 2,1 mg/dl, Na 135 meq/l, K 3,1 meq/l, Cl 117 meq/l. Orina con pH alcalino. Respecto al daño renal en este paciente es cierto que: 1. No habrá respuesta a la administración de bicarbonato oral, siendo preciso infusión de bicarbonato i.v. 2. Se caracteriza por la presencia de hipocalciuria.

-22-

booksmedicos.org 3. Es característico encontrar hipocitraturia. 4. Aunque el paciente tenga buena respuesta al tratamiento con melfalán prednisona el daño renal será irreversible. 5. Estaría contraindicado administrar suplementos de potasio oral.

115.

2. Manejo conservador, el paciente no es candidato a diálisis por su patología cardiaca. 3. Dado que el paciente no es diabético, estaría indicado manejo conservador hasta que llegue a Clcr 10 ml/min, en ese momento deberá iniciar diálisis. 4. La opción más razonable es valorar un trasplante de vivo. 5. Debemos iniciar programa de diálisis crónica por uremia en paciente con insuficiencia renal avanzada.

Paciente mujer de 78 años de edad. AP de HTA, DL, DM II de más de 20 años de evolución y nefropatía diabética con proteinuria de 2,8 g/día en el último control. Sigue tratamiento con losartán, enalapril, amlodipino, atorvastatina e insulina ultralenta. En analítica de control aparece hiperpotasemia de 6,2 meq/l sin alteraciones en el electrocardiograma. ¿Cuál de las opciones siguientes le parece más correcta?

117.

1. Hiperpotasemia secundaria a tratamiento con IECA. Suspender enalapril, asociar resinas de intercambio iónico y realizar consejo dietético. 2. Hiperpotasemia por acidosis tubular de tipo IV. Mantener tratamiento y realizar consejo dietético para restringir la ingesta de potasio. 3. Hiperpotasemia por AT tipo IV e ingesta de fármacos bloqueantes del SRAA. Estaría indicado suspender losartán y enalapril, además de restringir la ingesta de potasio e iniciar tratamiento con resinas de intercambio iónico a dosis altas. 4. Hiperpotasemia por AT tipo IV e ingesta de fármacos bloqueantes del SRAA. Estaría indicado suspender amlodipino y enalapril, además de restringir la ingesta de potasio e iniciar tratamiento con resinas de intercambio iónico a dosis altas. 5. Hiperpotasemia por AT tipo IV e ingesta de fármacos bloqueantes del SRAA. Estaría indicado suspender temporalmente enalapril o losartán, restringir la ingesta de potasio e iniciar tratamiento con resinas de intercambio iónico a dosis bajas.

116.

SIMULACRO 5

Un varón de 70 años, con antecedentes de ictus isquémico, acude a urgencias por crisis hipertensiva de 200/100 mmHg. AS: Cr 1.3 mg/dL, urea 30 mg/dL, pH 7.46, bicarbonato 30. El médico que le atiende le pauta un comprimido de captopril con lo que baja la tensión a 140/60 mmHg, pero en una segunda analítica presenta Cr 4.5 mg/dL. ¿Cuál es la primera patología a descartar? 1. 2. 3. 4. 5.

118.

GN membranosa. NTA. Estenosis de arteria renal. NIIA. Reacción normal tras bajar la tensión.

Papá y mamá acuden muy ansiosos a tu consulta porque su hijo mediano, de 4 años, todavía se orina de noche en la cama. Refieren que su hija mayor mostró control completo de los esfínteres desde los 2 años y medio y les preocupa que éste se esté retrasando así. Durante el día no presenta fugas y realiza un ciclo miccional normal. Controla perfectamente las heces. A pesar de que no refiere disuria, su pediatra le ha realizado varios cultivos de orina que han resultado negativos. 1. Se trata de una situación fisiológica y se deberá tranquilizar a los padres. 2. Se trata de una enuresis nocturna y se deberá iniciar tratamiento con anticolinérgicos nocturnos. 3. Se trata de un síndrome enurético y se deberá realizar un estudio urodinámico. 4. Se trata de una enuresis nocturna y se deberá iniciar tratamiento con hormona antidiurética por la noche. 5. Se trata de un síndrome enurético y se deberá remitir al paciente y a su familia al completo a psiquiatría para iniciar una terapia conductual.

Paciente varón de 66 años de edad con AP de sobrepeso, intolerancia hidrocarbonada, HTA, ERC secundaria a nefroangioesclerosis y enfermedad coronaria con enfermedad de tres vasos revascularizada. Vive solo y es escritor. Tras último IAM quedó con una FEVI del 15%. Es seguido en la consulta de ERCA (enfermedad renal crónica avanzada) porque presenta urea 190 mg/dl, Cr 3,5 y ClCr de 16 ml/min. A lo largo de los últimos meses ha presentado varios ingresos por ICC descompensada y, a pesar de tratamiento diurético intensivo, mantiene clase funcional IV (NYHA). Estaría indicado: 1. Debemos iniciar programa de diálisis crónica lo antes posible por sobrecarga de volumen con mala respuesta a diuréticos.

119.

-23-

Un paciente de 52 años acude al servicio de urgencias por fiebre elevada, dolor perineal y malestar general. Como antecedentes de interés refiere la realización

booksmedicos.org

por un cuadro de hematuria autolimitada. Se le realiza una ecografía, donde se aprecia una lesión ocupante de espacio intravesical de unos 2 cm de diámetro, pediculada. En este caso:

de una BTRE esa misma mañana. A la exploración presenta hipotensión mantenida, leucocitosis con desviación izquierda y 39,5º C. No es correcto: 1. Es fundamental la estabilización hemodinámica del paciente, que puede encontrarse seriamente afectada en estos casos. 2. Tras la BTRE es excepcional la aparición de cuadros como el de este paciente, ya que no es en absoluto una de las complicaciones más frecuentes de dicho procedimiento. 3. Precisará tratamiento antibiótico prolongado. 4. No es probable que precise una intervención quirúrgica para la resolución del cuadro clínico. 5. Deberá iniciarse tratamiento i.v. con un antimicrobiano de amplio espectro.

120.

1. Lo primero a realizar deberá ser una toma de biopsia para distinguir si es superficial o invasivo. 2. Si la citología es positiva será un factor de mal pronóstico. 3. Antes de plantear ningún tipo de actuación terapéutica será necesaria la realización de un estudio de extensión. 4. Se debe proponer una cistectomía radical a este paciente, ya que la probabilidades de tener un tumor infiltrante son muy altas. 5. Un ciclo de BCG previa la cirugía mejorará significativamente la esperanza de vida de este paciente.

Una paciente de 23 años, sexualmente activa, presenta síntomas compatibles con una infección de orina, la cuarta en el último trimestre. Las tres anteriores se han tratado adecuadamente con quinolonas, aunque en ningún momento se han realizado urocultivos. Señale la falsa:

123.

1. Es posible que se trate de una infección por Staphylococo saprophyticus. 2. Sería conveniente la obtención de muestras para urocultivo. 3. Sería conveniente revisar a la paciente para valorar la necesidad de un estudio urológico completo. 4. Sería conveniente tratar a la pareja sexual para eliminar el ciclo infeccioso. 5. Sería conveniente recomendar una serie de medidas higiénico-dietéticas a la paciente.

Un hombre de 61 años, hipertenso bien controlado y con DM tipo 2, acude a su consulta porque en una ecografía realizada para valorar su dispepsia habitual se le ha detectado una masa renal. Se trata de una masa de 3 cm, bien definida, con bordes lisos, de contenido hipoecoico homogéneo y con refuerzo ecográfico posterior. Su actitud será: 125.

Tumor de células de Sertoli. Carcinoma embrionario. Seminoma puro. Teratoma maduro. Coriocarcinoma.

La consecución del más alto nivel de calidad con una cantidad fija de recursos es la: 1. 2. 3. 4. 5.

1. Tranquilizar al paciente y realizar una ecografía anual de control, ya que parece tratarse de un quiste simple. 2. Solicitar una TAC abdominal de forma inmediata. 3. Solicitar una PAAF de la masa. 4. Programar una nefrectomía parcial. 5. Derivar al paciente a oncología.

122.

Varón de 35 años acude por bulto en testículo de reciente aparición, pétreo no doloroso. En ecografía se objetiva una lesión intraparenquimatosa hipoecoica de 2 cm. Los marcadores tumorales son normales. Se realiza orquiectomía radical. Usted espera encontrar: 1. 2. 3. 4. 5.

124. 121.

SIMULACRO 5

Eficacia. Efectividad. Eficiencia. Optimización. Idoneidad.

Señale la FALSA en relación con los estudios de prevalencia: 1. Son estudios muy útiles en la planificación sanitaria y para la descripción de características de poblaciones. 2. Son útiles en el estudio de enfermedades crónicas y poco frecuentes. 3. No son útiles para el estudio de enfermedades de corta duración.

Un paciente de 76 años acude a su consulta derivado desde el servicio de urgencia, a donde acudió

-24-

booksmedicos.org 4. No permiten conocer la secuencia temporal entre el factor de estudio y la enfermedad. 5. No pueden distinguir entre factores de riesgo de la enfermedad y factores pronósticos de la misma.

126.

Programa de rehabilitación postinfarto. Autoexploración mamaria. Frotis de Papanicolaou en mujeres > 20 años. Fluoración del agua. Realización de placa de tórax en individuos tuberculín-positivos.

131.

132.

Un estudio epidemiológico inferencial concluye que existe una asociación estadísticamente significativa entre un factor de riesgo y cierta enfermedad, con un nivel de significación p < 0,05. ¿Cuál es la interpretación correcta de este resultado?

Indique la prueba de significación estadística a utilizar en la comparación de medias de dos muestras independientes:

El screening es una actividad de: 1. 2. 3. 4. 5.

1. 2. 3. 4. 5.

Prevención primaria. Prevención secundaria. Prevención terciaria. Promoción de la salud. Prevención primaria y promoción. 133.

129.

Los factores epidemiológicos. Los factores sociodemográficos y culturales. La disponibilidad de recursos. La eficiencia del sistema. La accesibilidad geográfica.

1. En caso de no existir diferencia entre expuestos y no expuestos al factor de riesgo, la probabilidad de observar por azar los resultados obtenidos es menor del 5%. 2. Se puede concluir que la asociación real existe, ya que hemos obtenido significación estadística. 3. Hasta el 95% de los expuestos al factor de riesgo desarrollarán la enfermedad. 4. Menos de un 5% de expuestos al factor de riesgo desarrollarán la enfermedad. 5. La existencia de una asociación estadísticamente significativa confirma la hipótesis de una relación causal entre el factor de riesgo y la enfermedad estudiada.

Para la elección de un programa de detección precoz de enfermedad se deben tener en cuenta una serie de criterios, entre los cuales NO figura: 1. Debe conocerse la historia natural de la enfermedad. 2. El coste del test debe ser razonable dentro del presupuesto de salud. 3. El tratamiento en el estado presintomático debe reducir la morbimortalidad en mayor medida que el tratamiento en estado sintomático. 4. La enfermedad debe ser poco frecuente y grave. 5. La comunidad debe sentir la necesidad de programas de salud pública.

128.

Entre los factores que determinan el grado de utilización de los servicios de salud no se encuentran: 1. 2. 3. 4. 5.

¿Cuál de estas medidas podría encuadrarse en un programa de prevención primaria? 1. 2. 3. 4. 5.

127.

130.

SIMULACRO 5

t de Student. Chi-cuadrado. Test exacto de Fisher. Análisis de la varianza. Coeficiente de correlación de Pearson.

Las enfermedades cuarentenables son:

Respecto a la epidemiología del cáncer de cuello uterino señalar la ERRÓNEA: 1. 2. 3. 4. 5.

1. Es más frecuente su aparición en nulíparas que en multíparas. 2. Es más frecuente en mujeres que tienen su primer hijo a edades más jóvenes. 3. Es más frecuente en casadas. 4. Se ha relacionado con infecciones virales. 5. No se ha visto relación con contracepción hormonal.

134.

-25-

Fiebre amarilla, cólera, tifus. Tifus, peste, cólera. Fiebre amarilla, cólera, peste. Cólera, rabia, peste. Rabia, peste, fiebre amarilla.

Señale la sentencia FALSA en relación con la vacuna del Haemophilus influenzae tipo b (Hib):

booksmedicos.org 1. Se asocia a la DTP en el calendario vacunal. 2. Disminuye la tasa de meningitis por Haemophilus influenzae tipo b y meningococo serotipo B. 3. No es precisa en los mayores de 5 años. 4. No posee contraindicaciones específicas. 5. Disminuye las tasas de portadores crónicos.

1. 2. 3. 4. 5.

139. 135.

¿Cuál es el denominador de la razón de dependencia? 1. 2. 3. 4. 5.

136.

140.

Oxigenoterapia y broncodilatadores vía oral. Oxigenoterapia y corticoides sistémicos. Broncodilatadores inhalados. Opiáceos y benzodiacepinas. Diuréticos, broncodilatadores y oxígeno.

El cuadro confusional agudo en el paciente oncológico se va a producir como consecuencia de la claudicación mental del paciente, cuál de las siguientes posibilidades de tratamiento NO es correcta:

Cuando hablamos de pacientes subsidiarios de recibir cuidados paliativos, ¿cuál de los siguientes criterios no definiría a un paciente oncológico terminal?

Un paciente de 56 años con antecedente de IAM a los 48 años con enfermedad de 2 vasos y portador de 2 stent se encuentra ingresado por epistaxis posterior severa por la fosa nasal derecha. Se llevaron a cabo varios taponamientos anteriores que no controlaron el sangrado, por lo que se colocó un taponamiento posterior y se ingresó al paciente en la UVI para monitorización. A las 48 h comienza con nuevo sangrado por la fosa derecha. Descartada la embolización al no contar con Servicio de Radiología Intervencionista, a la hora de plantear un tratamiento quirúrgico para la epistaxis, ¿cuál de los siguientes sería el de elección? 1. Ligadura de la arteria carótida externa por cervicotomía lateral. 2. Ligadura de la arteria esfenopalatina por vía endoscópica. 3. Ligadura de la arteria etmoidal anterior por vía de etmoidectomía externa. 4. Ligadura de la arteria maxilar interna mediante abordaje tipo Cadwell-Luc. 5. Ligadura de la arteria angular por vía externa.

1. Si aparecen alucinaciones o síntomas psicóticos, podemos emplear haloperidol. 2. No es una buena alternativa al haloperidol en pacientes agitados o angustiados la clorpromacina, debido a su mayor poder de sedación. 3. Si la causa es farmacológica, suspenderemos o reduciremos la dosis del fármaco. 4. Si es por hipoxia o cianosis, podemos emplear la oxigenoterapia. 5. La tioridacina podría ser de elección en ancianos por su menor efecto sedante.

138.

Nalorfina. Pentazocina. Metadona. Naloxona. Meperidina.

1. Esperanza de vida limitada. 2. Presencia de síntomas que requieren manejo específico. 3. Complicaciones irreversibles en progresión. 4. Enfermedad oncológica en fase estable o de secuelas. 5. Los tratamientos con finalidad curativa no están indicados.

Cuál de las siguientes pautas seleccionaría en un paciente de 69 años con un cáncer de pulmón en progresión con metástasis pulmonares bilaterales, esperanza de vida de 2-3 semanas y disnea de reposo como síntoma dominante: 1. 2. 3. 4. 5.

137.

Población total. Población mayor de 65 años o menor de 14. Población entre 15 y 64 años de edad. Población mayor de 14 años. Población menor de 14 años.

SIMULACRO 5

141.

Señale aquel opioide que presenta una baja actividad intrínseca en receptores mu y alta sobre los sigma, implicando gran riesgo de provocar disforia, cuadros psicomotores y alucinaciones:

-26-

Una paciente de 75 años es intervenida tras una fractura de cadera con implantación de prótesis completa en articulación coxofemoral izquierda. A los 2 meses aqueja dolor en región inguinal, glútea y cara lateral del muslo. Ocasionalmente, picos febriles aislados. El dolor se exacerba con la deambulación y progresivamente presenta impotencia funcional parcial. Señale la opción correcta:

booksmedicos.org

bras musculares e infiltrados inflamatorios linfocitarios. ¿Qué diagnóstico le parece más probable en esta paciente?

1. La infección de una prótesis rara vez cursa de forma indolente con síntomas leves y es difícil que se retrase en el diagnóstico. 2. En el 80% de los casos se aísla el germen (gram +) en el líquido articular. 3. La cifra de glucosa es baja y el lactato alto en el líquido sinovial, siendo este dato muy específico. 4. La gammagrafía ósea es imprescindible y sumamente útil en el diagnóstico de la artitis séptica de cadera, especialmente en fases tempranas. 5. Es de considerar la administración intraarticular de antibióticos a altas dosis para evitar posteriores secuelas.

1. Déficit de miofosforilasa (enfermedad de Mc Ardle). 2. Miastenia gravis. 3. Distrofia muscular de Duchenne. 4. Polimiositis. 5. Fibromialgia.

145. 142.

Un paciente de 39 años con insuficiencia renal crónica, en tratamiento con hemodiálisis desde hace 12 años, consulta por presentar poliartritis simétrica de hombros, carpos y rodillas y síndrome de túnel carpiano bilateral confirmado en estudio neurofisiológico. El recuento celular del líquido sinovial muestra 100 células/mm3. ¿Cuál de las siguientes considera que es la causa más probable de su artritis?

146.

144.

Mujer de 65 años de edad sin síntomas articulares hasta hace 5 años. Desde entonces, de forma intermitente, refiere dolor en interfalángicas proximales y distales de ambas manos con signos inflamatorios de corta duración seguidos de desarrollo de tumefacción de consistencia ósea y localización asimétrica alrededor de la articulación. ¿Cuál es el diagnóstico más probable? 1. Artrosis. 2. Enfermedad por depósitos de pirofosfato cálcico dihidratado. 3. Artritis reumatoide. 4. Hiperparatiroidismo. 5. Hemocromatosis.

1. Artritis por depósitos de pirofostato cálcico. 2. Amiloidosis por depósito de beta-2 microglobulina. 3. Gota úrica poliarticular. 4. Osteodistrofia renal. 5. Artritis reactiva postinfecciosa.

143.

SIMULACRO 5

Varón de 32 años que acude a consulta por la aparición de una parótida aumentada de tamaño, boca seca y queratitis seca con un test de Schirmmer positivo (ausencia de secreción lacrimal). En la biopsia parotídea se observa un infiltrado linfocitario con predominio CD8. Como antecedente de interés refiere un cuadro mononucleósico hace 2 meses. ¿Qué es lo más probable?

Mujer de 50 años con diabetes mellitus tipo 1 desde hace 35. Al bajar de una acera tiene una torcedura de su tobillo derecho con mucha inflamación pero escaso dolor. Un mes más tarde el tobillo sigue hinchado y apenas le produce molestias. Al explorar se demuestra movilidad aumentada en el tobillo pese al aumento de volumen y no se obtienen reflejos osteotendinosos en piernas con clara hipoestesia en pies y disminución de sensibilidad vibratoria en piernas. ¿Cuál será su diagnóstico más probable?

1. 2. 3. 4. 5.

1. 2. 3. 4. 5.

Sarcoidosis. Linfoma linfoblástico. Sínd. de linfocitosis difusa infiltrativa por HIV. Sínd. de Sjögren postinfeccioso. Recidiva mononucleósica.

Una mujer de 45 años consulta por presentar debilidad muscular en las cinturas escapular y pelviana de 2 meses de evolución. La analítica en sangre muestra un valor de CPK de 2.520 U/l (valor normal < 200) y de transaminasas elevadas. Un estudio electromiográfico presenta potenciales de unidad motora de baja amplitud y polifásicos. Una biopsia muscular muestra necrosis de las fi-

147.

Artritis séptica. Artropatía neuropática. Artritis microcristalina. Distrofia simpática refleja. Fractura de maléolo.

Un liquido sinovial de aspecto xantocrómico, con recuento leucocitario menor de 2.000/mm3, y polimorfonucleares inferiores a un 25% y sin disminución de la concentración de glucosa, es indicativo de: 1. Artritis reumatoide. 2. Gota úrica.

-27-

booksmedicos.org 3. Artritis séptica. 4. Artritis traumática. 5. Pseudogota.

148.

2. El urato se sintetiza en tejidos que contienen xantina-oxidasa, sobre todo hígado e intestino delgado. 3. La mayoría de las personas con hiperuricemia primaria presentan un trastorno en el control renal del ácido úrico. 4. La cetoacidosis diabética, el ayuno, intoxicación por etanol o salicilatos y la acidosis láctica son factores coadyuvantes a disminuir la excreción de ácido úrico. 5. La creatinina plasmática, el nitrógeno ureico y la concentración de urato en plasma se correlacionan de forma estrecha.

Una mujer de 25 años consulta por episodios de ortopnea con estridor; así como pérdida de fuerza en pierna derecha y sensación de acorchamiento en 4º y 5º dedos de la mano izquierda. La espirometría demuestra obstrucción al flujo en vías aéreas superiores. En la analítica destacan anemia, VSG elevada, y hematuria microscópica con cilindros, así como c-ANCAs elevados. ¿Cuál es la enfermedad más probable? 151. 1. 2. 3. 4. 5.

149.

150.

Enfermedad de Churg-Strauss. Panarteritis nodosa. Granulomatosis de Wegener. Tuberculosis. Granulomatosis linfomatoide.

Una mujer de 45 años consulta por fiebre, disnea, tos, rash cutáneo, diarrea y disminución de sensibilidad en la pierna derecha, de dos meses de duración. Tiene antecedentes de rinitis y asma con control difícil desde hace 15 años; sigue tratamiento con corticoesteroides orales frecuentemente, con buena respuesta. En la exploración presenta nódulos subcutáneos en superficies extensoras de miembros superiores, sibilancias en la auscultación respiratoria y déficit sensitivo en pierna derecha. En el hemograma presenta: hematocrito 36%, leucocitos 12.500 con un 38% de eosinófilos; la función renal y el análisis de orina son normales. En la radiografía de tórax se aprecian infiltrados alveolares parcheados. ¿Cuál de los siguientes diagnósticos es el más probable? 1. 2. 3. 4. 5.

SIMULACRO 5

Una mujer de 23 años de edad con examen clínico normal el año previo, debuta con clínica de 6 semanas de evolución de astenia progresiva y aumento de 5 kg de peso, edemas, HTA, hematuria, cilindros hemáticos, proteinuria (> 6 g/día), hipoalbuminemia y aumento del colesterol. La creatinina sérica es de 2,9 mg/dl. Los ANA y anti-DNA son positivos a títulos altos, junto a un descenso de los niveles del complemento. En la biopsia renal, señale cuál es el hallazgo más esperable: 1. 2. 3. 4. 5.

152.

Granulomatosis alérgica de Churg-Strauss. Neumonía eosinofíla crónica. Síndrome hipereosinófilo. Granulomatosis linfomatoide. Poliarteritis nodosa.

Un paciente de 35 años lleva padeciendo artralgias en las articulaciones metacarpofalángicas (MCF), muñecas y tobillos cuatro meses. Posteriormente desarrolla artritis en MCF, interfalángicas proximales de manos, metatarsofalángicas, muñecas y tobillos, acompañadas de rigidez matutina de más de 3 horas de duración. ¿Cuál es el diagnóstico más probable? 1. 2. 3. 4. 5.

Un hombre de 63 años de edad refiere episodios recurrentes de artritis inflamatoria en articulación metatarsofalángica del primer dedo del pie derecho. En otras ocasiones se presenta en metacarpofalángica del pulgar de la mano izquierda. El paciente refiere ingesta frecuente de alcohol. Señale la respuesta FALSA:

153.

1. Es probable que el paciente presente concentraciones de urato en plasma superiores a 7 mg/dl.

-28-

Glomerulonefritis proliferativa difusa. Nefritis lúpica proliferativa focal. Nefropatía membranosa. Glomeruloesclerosis focal y segmentaria. Vasculitis necrotizante sistémica.

Artrosis generalizada. Reumatismo poliarticular agudo (fiebre reumática). Gota poliarticular. Artritis reumatoide. Enfermedad de Whipple.

Un paciente de 27 años viene a la consulta con una historia de 1 año de duración de dolor en ambos glúteos irradiado por cara posterior de muslo hasta su porción media. El dolor es peor al amanececer; se acompaña de rigidez lumbar y mejora con el ejercicio. ¿Cuál de los siguientes procedimientos aportaría más información diagnóstica en este paciente?

booksmedicos.org 1. La velocidad de sedimentación y los niveles de proteína C reactiva. 2. Los niveles de proteína C reactiva y de antiestreptolisina O (ASLO). 3. La resonancia magnética de columna lumbar. 4. El tipaje HLA-B27. 5. La radiografía de las articulaciones sacroilíacas.

3. Reducción cerrada y aplicación de yeso. 4. Exploración del nervio radial y enclavado endomedular bloqueado. 5. Estudio electrodiagnóstico del nervio radial.

157. 154.

Paciente de 70 años, sin antecedentes de interés, se presenta en Urgencias por la aparición de un “bulto” al nivel del tercio superior del muslo derecho. La exploración demuestra una deformidad en “sable” a nivel de las tibias, temperatura cutánea elevada a nivel de ambas piernas y aumento marcado de volumen del muslo derecho. Presenta una calcemia y fosfatemia normal y una actividad acentuada de la fosfatasa alcalina. La exploración por RM y la biopsia del muslo demuestra una afectación maligna, ¿cuál sería el tumor más probable en este caso? 1. 2. 3. 4. 5.

155.

156.

¿Cuál de los siguientes nervios es el más frecuentemente lesionado cuando se obtiene injerto óseo de la región más posterior de la cresta ilíaca? 1. 2. 3. 4. 5.

158.

Condrosarcoma. Fibrosarcoma. Osteosarcoma. Linfoma. Mieloma múltiple.

N. femorocutáneo lateral. N. glúteo superior. Nervio cluneal. Raíz de L5. Nervio obturador.

¿Cuál de las siguientes estructuras forman los límites de la tabaquera anatómica en la muñeca? 1. Extensor largo del pulgar y abductor largo del pulgar. 2. Abductor corto del pulgar y abductor largo del pulgar. 3. Extensor largo del pulgar y extensor corto del pulgar. 4. Arteria radial y extensor corto del pulgar. 5. Abductor largo del pulgar y extensor corto del pulgar.

Ante una paciente de 40 años de edad que presenta un cuadro caracterizado por Raynaud, poliartritis no deformante, lesiones cutáneas eritematosas en mejillas y debilidad muscular proximal y cuyo dato de laboratorio más importante es la presencia de títulos altos de anticuerpos anti-RNP, la primera sospecha diagnóstica debe ser: 1. 2. 3. 4. 5.

SIMULACRO 5

159.

AR (artritis reumatoide). LES (lupus eritematoso sistémico). Esclerodermia. Polimiositis. EMTC (enfermedad mixta del tejido conectivo).

Una mujer de 32 años con lupus eritematoso sistémico tratada con metotrexate y corticosteroides orales acude a nuestra consulta referiendo dolor inguinal derecho con la deambulación y dolor nocturno. El examen revela dolor con la rotación interna y externa, estando la flexión limitada a 105º debido al disconfort con dicho movimiento. Los estudios de laboratorio muestran unos leucocitos en sangre de 9.000/mm3 y una velocidad de sedimentación globular de 35 mm/h. Nuestro estudio debería incluir a continuación: 1. 2. 3. 4. 5.

Un hombre de 37 años sufre una fractura de tercio medio de húmero derecho tras ser golpeado por un coche. La radiología muestra una fractura oblicua con tercer fragmento en alas de mariposa así como una limitación a la extension de muñeca, dedos y el pulgar durante la exploracion física. El manejo de dicha fractura debería consistir en: 160. 1. Osteosíntesis con placa mediante abordaje anterolateral. 2. Fijación externa.

-29-

Examen bajo fluoroscopia. RNM. Gammagrafía ósea. Artrografía. Aspiración y artrografía.

Un bebé de 2 semanas nos ha sido referido para evaluación por nuestra parte por ausencia de movimiento de cadera izquierda. La historia clínica muestra que el paciente nació prematuro 6 sema-

booksmedicos.org 1. 2. 3. 4. 5.

nas por cesárea. El examen revela ausencia de fiebre, y hay una discreta inflamación en el muslo. El movimiento pasivo de la cadera aparece aumentar el dolor de la cadera así como su muy limitada movilidad. Una radiografía de la pelvis muestra subluxación media de la cadera izquierda. El próximo paso en la evaluación consiste en: 164. 1. 2. 3. 4. 5.

161.

Aspiración de la cadera izquierda. Aplicación de Arnés de Pavlik. Gammagrafía con galio. RNM de la columna. Tracción modificada de Bryant.

Enfermo de 55 años que refiere astenia de presentación reciente y molestias torácicas inespecíficas desde hace más tiempo. En el hemograma las cifras de leucocitos y plaquetas son normales, pero presenta una hemoglobina de 5 g/dL con ausencia de reticulocitos. La radiografía de tórax muestra un ensanchamiento mediastínico. El diagnóstico más probable es: 165.

162.

166. 1. 2. 3. 4. 5.

163.

Mieloptisis. Aplasia de médula ósea. Leucemia aguda mieloblástica. Cirrosis hepática con hiperesplenismo. Hemoglobinuria paroxística nocturna.

LDH sérica. Haptoglobina sérica. Niveles séricos de vitamina B12. Prueba de Ham. Niveles séricos de folato.

Niño de 5 años con historia de anemia crónica que se agrava con procesos infecciosos. En la exploración se aprecia ligera ictericia y esplenomegalia. Acude por fiebre y empeoramiento de la astenia, por lo que comienza tratamiento antibiótico. El hemograma presenta leucocitos 9.5 x 109/L, hemoglobina 5 g/dL, reticulocitos ausentes, plaquetas 150 x 109/L. ¿Cuál es la explicación más probable? 1. 2. 3. 4. 5.

Enfermo de 65 años, fumador y bebedor habitual, que consulta por astenia de semanas de evolución, progresiva, por lo que se realiza un hemograma con los siguientes resultados: leucocitos 18 x 109/L con formas inmaduras, hemoglobina 7 g/dL, plaquetas 80 x 109/L. En el frotis se aprecian hematíes en forma de lágrima y normoblastos. ¿Cuál es la explicación más probable de dichos hallazgos?

Hipotiroidismo. Síndrome mielodisplásico. Anemia inmunohemolítica. Aplasia de médula ósea. Anemia megaloblástica.

Enfermo de 45 años que refiere debilidad crónica acompañada de episodios de orinas oscuras seguidos de incremento de la debilidad. En la exploración se observa ictericia conjuntival. El hemograma refleja una hemoglobina de 7 g/dL con reticulocitos de 35 x 109/L. Entre las siguientes pruebas complementarias, ¿cuál es la menos indicada para el estudio del paciente? 1. 2. 3. 4. 5.

1. Linfoma linfoblástico T. 2. Timoma con eritroblastopenia. 3. Carcinoma microcítico pulmonar metastásico a médula ósea. 4. Mieloptisis. 5. Enfermedad de Hodgkin con afectación medular.

SIMULACRO 5

Infección por parvovirus. Crisis hemolítica por fármacos. Crisis hemolítica asociada a la infección. Deficiencia de folato. Deficencia de hierro.

Paciente de 78 años que consulta por debilidad y astenia progresivas en los últimos meses. El hemograma presenta los siguientes datos: leucocitos 4.2 x 109/L, hemoglobina 8 g/dL, VCM 105 fL, plaquetas 80 x 109/L, frotis con granulocitos de núcleos bilobulados. En relación con el diagnóstico más probable del paciente, ¿qué respuesta es falsa? 1. Es posible un aumento del porcentaje de blastos en la médula ósea. 2. Es necesario un estudio citogenético de la médula ósea. 3. Será útil el hierro en el tratamiento. 4. Es improbable un hallazgo citogenético del 5q. 5. La celularidad de médula ósea probablemente se encuentra incrementada.

Mujer de 45 años que consulta por astenia y debilidad y sensación de mareo con ortostatismo. El hemograma es el siguiente: leucocitos 7.5 x 109/L con fórmula normal, hemoglobina 7 g/dL, VCM 105 fL, plaquetas 180 x 109/L, reticulocitos 150 x 109/L. ¿Cuál es el diagnóstico más probable entre los siguientes?

-30-

booksmedicos.org 167.

física se aprecia palidez cutánea y mucosa y las citadas petequias, sin palparse organomegalias. El hemograma tiene los siguientes resultados: hemoglobina 6.5 g/dL, leucocitos 3.8 x 109/L con 50% neutrófilos, 2% eosinófilos, 13% linfocitos, 35% blastos, plaquetas 15 x 109/L. El estudio citogenético de los blastos demuestra t(15;17). ¿Cuál de los siguientes fármacos está especialmente indicado en el tratamiento de la enfermedad?

Mujer de 55 años que consulta por cefaleas, acúfenos, mareos, disminución de peso y sudoración frecuente en las últimas semanas. En la exploración física se palpa esplenomegalia y el hemograma presenta un hematocrito de 56% y hemoglobina 17 g/dL. En relación al diagnóstico más probable de la paciente, es falso: 1. El valor de la eritropoyetina en suero será bajo. 2. La médula ósea presentará panmielosis. 3. La fosfatasa alcalina leucocitaria estará disminuida. 4. El prurito es característico en la enfermedad. 5. Existirá crecimiento endógeno de colonias eritroides.

168.

1. 2. 3. 4. 5.

171.

Mujer de 45 años que presenta deterioro general y astenia en los últimos meses. En la exploración física se palpa esplenomegalia moderada. El hemograma tiene leucocitos 34 x 109/L con 80% neutrófilos, 5% cayados 1% metamielocitos, 2% mielocitos, 1% basófilos, 10% linfocitos, 1% monocitos, hemoglobina 10 g/dL, plaquetas 480 x 109/L. El estudio citogenético es normal y se detecta reordenamiento BCR/ABL. ¿Cuál es el tratamiento de primera línea para la paciente? 1. 2. 3. 4. 5.

Hidroxiurea. Mesilato de imatinib. Interferón alfa. Autotrasplante de progenitores hematopoyéticos. Alotrasplante de progenitores hematopoyéticos.

Varón de 70 años, asintomático, remitido para estudio de hemograma patológico con los siguientes hallazgos: hemoglobina 12 g/dL, plaquetas 180 x109/L, leucocitos 20 x 109/L, con 90% linfocitos pequeños CD19, CD20, CD5, IgM lambda e IgD lambda. En la exploración presenta adenopatías cervicales bilaterales. ¿Cuál es el diagnóstico del paciente? 1. 2. 3. 4. 5.

170.

Imatinib. Fludarabina. Cladribina. Rituximab. Acido transretinoico.

Sobre la infección por HIV en la infancia, es FALSO: 1. Todos los niños nacidos de madres seropositivas tendrán al nacer anticuerpos frente a HIV positivos. 2. La transmisión vertical ocurre sólo durante el parto. 3. Anticuerpos maternos frente a HIV persisten en el recién nacido alrededor de diez meses. 4. Transmisión del HIV postnatal puede ocurrir por lactancia materna. 5. Los niños infectados por HIV deben ser vacunados de forma rutinaria cuando esté indicado.

172. 169.

SIMULACRO 5

¿Cuál de los siguientes es el signo más característico de la rubéola o sarampión alemán? 1. 2. 3. 4.

Enantema de Forcheimer. Manchas de Koplik. Lengua en fresa blanca. Amígdalas cubiertas por un exudado blancogrisáceo. 5. Adenopatías retroauriculares y occipitales.

Leucemia linfática crónica estadio I de Rai. Leucemia linfática crónica estadio II de Rai. Fase leucémica de enfermedad de Hodgkin. Tricoleucemia. Linfoma no hodgkiniano.

173.

Enfermo de 65 años que consulta por debilidad intensa de evolución rápida y sangrado fácil al cepillarse los dientes, además de aparición de petequias en miembros inferiores. En la exploración

Una niña recién nacida a término, de adecuado peso, con un Apgar de 9/10, a las 4 horas de vida no ha realizado diuresis, y su creatinina sanguínea es de 1 mg/dl. Señale la actitud correcta de manejo: 1. 2. 3. 4.

-31-

Administrar furosemida. Realizar ecografía renal y de vías urinarias. Sondaje vesical y medición horaria de diuresis. Controlar niveles de creatinina, BUN y potasio.

booksmedicos.org 5. Esperar, ya que no existen datos patológicos en ese momento.

174.

1. Debe operarse urgentemente al realizar el diagnóstico, sobre todo si la cianosis y el compromiso hemodinámico son severos, sin esperar a la estabilización del recién nacido. 2. El neumotórax y la hipertensión pulmonar con persistencia de circulación fetal son complicaciones frecuentes. 3. Es más frecuente en el lado izquierdo. 4. Puede presentarse después del primer mes de vida en forma de vómitos, estreñimiento o dolor abdominal, e incluso detectarse de forma casual al realizar una radiografía de tórax en los casos leves. 5. Puede realizarse diagnóstico prenatal por ecografía, debiendo diferenciarse de la malformación adenomatoidea quística y del enfisema lobar congénito.

¿Cuándo recomienda la vacunación triple vírica la Asociación Española de Pediatría? 1. 2. 3. 4.

A los 15 meses, y en las niñas, a los 11-12 años. A los 15 meses, y en ambos sexos, a los 3-6 años. En ambos sexos a los 11-12 años. A los 9 meses, a los 15 meses y a los 11-12 años en las niñas. 5. A los 15 meses exclusivamente.

175.

178.

Un recién nacido a término, de adecuado peso, es atendido en la sala de partos. Nace impregnado de líquido amniótico espeso, presenta hipotonía, bradicardia severa y apnea. Señale la medida inicial de elección en su reanimación: 1. 2. 3. 4.

Intubación y ventilación manual. Intubación y aspiración de la vía respiratoria. Aspiración orofaríngea y ventilación con bolsa. Aspiración orofaríngea y estimulación manual de la respiración. 5. Aspiración orofaríngea y masaje cardíaco.

176.

177.

Neonato de 2 días de vida con perímetro craneal de 30 cm, varón, procedente de un embarazo mal controlado, con edad gestacional de 38 semanas y peso de 2400 g, adenopatías laterocervicales e inguinales, con fontanela de 4x4 cm, hepatoesplenomegalia y una opacidad en la pupila de ambos ojos, soplo sistólico en 2º EII. La madre recuerda que tuvo un proceso febril en los primeros meses. ¿Qué patología sospecha en este niño? 1. 2. 3. 4. 5.

Recién nacido pretérmino, de 1.100 g al nacimiento, que comienza a los 2 días de vida con crisis convulsivas clónicas erráticas. Los estudios analíticos son normales y en la ecografía cerebral se observa un hematoma en plexos coroideos con sangre intraventricular, leve hiperecogenicidad subependimaria y tamaño ventricular normal. Esta complicación es: 1. 2. 3. 4. 5.

SIMULACRO 5

179.

Hemorragia intraventricular grado II. Hemorragia interventricular grado III. Hemorragia interventricular grado IV. Leucomalacia periventricular. Encefalopatía hipóxico-isquémica.

Paciente de 16 años que acude a Urgencias por disminución del nivel de conciencia y confusión. La paciente tiene antecedentes de epilepsia en tratamiento médico y la familia dice que en el día de hoy ha tomado una dosis extra de dicho medicamento. En la exploración presenta marcha atáxica, nistagmo, e hirsutismo. ¿Qué medicación estará tomando? 1. 2. 3. 4. 5.

Le avisan con urgencia para valorar a un recién nacido postérmino, con antecedentes de sufrimiento fetal, que ha empeorado tras su primera toma de leche. En la exploración está cianótico, polipneico con Silverman de 4, abdomen excavado, hipoventilación de pulmón izquierdo y latido cardíaco desplazado a la derecha. Respecto a esta patología, ¿cuál es la INCORRECTA?

180.

-32-

Infección connatal por citomegalovirus. Sífilis congénita. Rubéola congénita. Infección intraparto por virus herpes simple. Toxoplasmosis congénita.

Fenitoína. Carbamacepina. Etosuximida. Acido valproico. Trimetadiona.

Paciente de 36 años, en proceso de duelo tras el fallecimiento de su marido, que con fines autolíticos ingiere una caja entera de doxepina. El ECG muestra un aumento del PR y del QRS, con depresión de la onda T. Las arritmias de esta paciente puede ser tratada con:

booksmedicos.org 1. 2. 3. 4. 5.

181.

182.

2. Las troponinas C, I y T son proteínas reguladoras que se hallan espaciadas a intervalos regulares sobre los filamentos finos. 3. En presencia de ATP, la actividad enzimática de la actina y la miosina provoca su entrecruzamiento cíclico. 4. En los filamentos gruesos, las porciones globulares de la miosina se proyectan hacia afuera para interaccionar con la actina. 5. El cAMP refuerza la fosforilación de la troponina I, proteína que acelera la relajación cardíaca.

Digital. Procainamida. Quinidina. Bicarbonato. Disopiramida.

¿Cuál de los siguientes fármacos debe ser utilizado con gran precaución por el riesgo derivado de su capacidad para inducir la actividad de varias de las isoenzimas CYP450 provocando interacciones peligrosas? 1. 2. 3. 4. 5.

Cloramfenicol. Eritromicina. Rifampicina. Ritonavir. Itraconazol.

185.

Con respecto a la refracción de los distintos medios ópticos del ojo, señale la respuesta correcta:

El factor de riesgo más importante para sufrir una neuropatía óptica isquémica anterior no arterítica es: 1. 2. 3. 4. 5.

184.

La determinación del punto del tórax en que se ausculta mejor un soplo cardíaco y las áreas a las que se irradia, pueden ser útiles para identificar la estructura cardíaca en que se origina. En relación a este tema, ¿cuál de las siguientes propuestas es ERRÓNEA? 1. Los soplos debidos a valvulopatías derechas se oyen mejor en el borde esternal derecho, entre el segundo y el cuarto espacio intercostal. 2. El soplo de la estenosis valvular aórtica suele ser máximo en el segundo espacio intercostal derecho y se irradia a las carótidas. 3. El soplo de regurgitación mitral suele ser máximo en la punta. 4. El soplo de regurgitación mitral se irradia al borde esternal izquierdo y la base del corazón cuando está predominantemente afectada la valva posterior. 5. Cuando está afectada fundamentalmente la valva anterior mitral, el soplo de regurgitación se irradia a la axila y la espalda.

1. La potencia dióptrica de la córnea en el ser humano es despreciable. 2. La córnea es la lente más potente del ojo. Su potencia es modulable gracias a la acción del músculo ciliar. 3. El cristalino es la lente más potente del ojo. Su acción es modulable gracias a la acción del músculo ciliar. 4. Cristalino y córnea tienen una potencia similar. 5. La córnea es más potente que el cristalino. Sin embargo, la potencia del cristalino es modulable gracias a la acción del músculo ciliar.

183.

SIMULACRO 5

186.

Hipercolesterolemia. Hipertrigliceridemia. Diabetes mellitus. HTA. Tener una papila óptica pequeña.

¿Qué proteína es codificada por la región preS2+S del genoma del VHB? 1. 2. 3. 4. 5.

Las células musculares estriadas del miocardio están compuestas de estructuras que se repiten de forma seriada, con miofibrillas de distintas proteínas. Señale la respuesta ERRÓNEA en relación a estos filamentos:

187.

1. En la contracción interaccionan entre sí los filamentos de miosina y de actina, de forma que los primeros se desplazan hacia la banda A.

HBcAg. HBeAg. Proteína principal. Proteína mediana. Proteína grande.

¿Qué tumor de los citados a continuación tiene su origen en células primitivas con potencialidad para desarrollarse siguiendo las líneas neuronal y neuroglial? 1. Hemangioblastoma. 2. Glioblastoma multiforme. 3. Craneofaringioma.

-33-

booksmedicos.org 4. Meduloblastoma. 5. Ependimoma.

188.

189.

1. Simule, de otro modo no es explicable esta mejoría. 2. Presente un glaucoma juvenil. Por ello hay que pedirle un campo visual. 3. Presente un defecto de refracción. Hay que graduarle. 4. Presente una catarata congénita. 5. Presente retinopatía diabética.

Un varón de 50 años, esplenectomizado por una PTI, presenta, tras ser mordido por un perro, un cuadro de fiebre, hipotensión, malestar general, leucocitosis, trombopenia y aumento de los tiempos de coagulación. Respecto a la entidad que presenta este paciente, ¿cuál es el tratamiento de elección? 1. 2. 3. 4. 5.

192.

Doxiciclina. Ciprofloxacino. Ceftazidima. Rifampicina. Penicilina.

193.

190.

191.

Realización de médula ósea. Sustitución de la fenitoína por carbamacepina. Biopsia de ganglio supraclavicular. Biopsia de ganglio cervical. Serología a VIH.

¿Por qué en la atelectasia NO suele ser importante la desaturación de O2? 1. 2. 3. 4. 5.

Agricultor que acude a urgencias con dificultad respiratoria y dolor cólico abdominal. Presenta cefalea, sin rigidez de nuca, pero con vértigo y temblores. Está miótico y comenta tener visión borrosa. No orina desde ayer por la mañana, pese a haber ingerido líquidos. Usted sospecha: 1. 2. 3. 4. 5.

Respecto a la fiebre botonosa mediterránea es FALSO que: 1. Es endémica en los países desarrollados. 2. Está causada por Rickettsia conorii, siendo transmitida por la garrapata del perro. 3. Clínicamente se manifiesta como una enfermedad aguda febril, con cefalea, artromialgias exantema maculopapuloso y una lesión de inoculación (mancha negra en el 75% de los pacientes). 4. En general, es una enfermedad benigna que cura sin secuelas. 5. Para tratarla, las tetraciclinas son los antibióticos de elección.

Un paciente de 46 años, diagnosticado de epilepsia tipo gran mal hace un año y en tratamiento con difenilhidantoína desarrolla, de forma progresiva, un cuadro de adenopatías múltiples, artralgias y fiebre de bajo grado. No refiere pérdida de peso y su estado general es bueno. Señale la actitud inicial más correcta: 1. 2. 3. 4. 5.

SIMULACRO 5

194.

Broncoconstricción. Hiperventilación. Circulación colateral. Vasoconstricción selectiva. Cortocircuito a nivel central.

Respecto al ciclo genital femenino es CIERTO que: 1. Consta de dos fases, proliferativa y secretora. 2. La fase proliferativa es constante. 3. A mitad de ciclo, el pico de LH es previo al pico de estrógenos y ocurre la ovulación. 4. Un ambiente androgénico excesivo en la capa granulosa produce atresia folicular. 5. La teca transforma los andrógenos de la granulosa en estradiol, mediante la aromatasa.

Intoxicación por paraquat. Intoxicación por organofosforados. Intoxicación por rodenticidas. Intoxicación por hidrocarburos aromáticos. Intoxicación por etilenglicol.

Paciente de 15 años de edad refiere ver peor la pizarra desde hace unos meses. La agudeza visual de lejos es 0,2 en su ojo derecho y 0,4 en su ojo izquierdo. Sin embargo, a través del agujero estenopeico, mejora a 1 en ambos ojos. Lo más probable es que:

195.

-34-

En el 90% de los carcinomas pulmonares de células pequeñas se puede encontrar una deleción del brazo corto de uno de los siguientes cromosomas. Señale cuál es:

booksmedicos.org 1. 2. 3. 4. 5.

196.

197.

la visita para conocer su opinión sobre la necesidad de un chequeo general y sobre la necesidad de realizar revisiones ginecológicas. Entre sus antecedentes personales destacan 3 gestaciones con partos eutócicos, menopausia hace dos años sin sangrado posterior. No fuma, no bebe, ni lo ha hecho nunca, no toma ningún medicamento, ni presenta ninguna patología activa o crónica conocida. No antecedentes familiares de interés. Última revisión ginecológica hace 18 años. ¿Qué actividades preventivas son las más recomendadas de inicio en esta mujer?

Cromosoma 13. Cromosoma 17. Cromosoma 3. Cromosoma 5. Cromosoma 7.

La siguiente es una intervención preventiva primaria de salud pública: 1. 2. 3. 4. 5.

Citología de cérvix. Mamografía rutinaria. Quimioterapia en pacientes con cáncer. Fisioterapia post accidente cerebro-vascular. Inmunización contra el sarampión.

1. Se trata de una mujer sana y por lo tanto no hay que hacer nada. 2. Citología vaginal cada 3 años, indicar búsqueda de atención médica si sangrado vaginal y mamografía bienal. 3. Cribado de HTA, hipercolesterolemia, obesidad, explorar actividad física y dieta, vacunación de difteria y tétanos o dosis de recuerdo cada diez años, citología vaginal anual en 2 años consecutivos, indicar búsqueda de atención médica si sangrado vaginal y mamografía bienal. 4. Anualmente medición de TA, de colesterol total, perfil tiroideo, densitometría, peso, talla, IMC, radiografía de tórax, explorar actividad física y dieta, vacunación de difteria y tétanos y dosis de recuerdo cada 10 años, citología vaginal anual en 2 años consecutivos, indicar búsqueda de atención médica si sangrado vaginal y mamografía bienal. 5. Medición de TA, de colesterol total, peso, talla e IMC, explorar actividad física y dieta, vacunación de difteria y tétanos y ecografía transvaginal, citologia vaginal y mamografía anuales.

La mortalidad materna se expresa en: 1. Número de muertes maternas por 10.000 embarazos. 2. Número de muertes maternas por 100.000 nacidos vivos. 3. Número de muertes maternas por 1.000 nacidos vivos. 4. Número de muertes maternas por 100.000 embarazos. 5. Número de muertes maternas por 1.000 embarazos.

198.

2007.17 Para vigilancia epidemiológica de la meningitis bacteriana, una persona que presenta fiebre y rigidez de nuca, podemos decir que es un: 1. 2. 3. 4. 5.

199.

200.

Caso confirmado. Caso probable. Caso sospechoso. Caso secundario. Caso identificado.

La Educación para la Salud de la población, de forma general constituye una estrategia de: 1. 2. 3. 4. 5.

SIMULACRO 5

Promoción de la salud. Protección de la salud. Prevención de la enfermedad. Potenciación de la salud. Restauración de la salud.

Acude a la consulta de su médico de familia una mujer de 54 años por cuadro catarral. Aprovecha

-35-

booksmedicos.org

booksmedicos.org

respuestas simulacro 5 Plantilla

NUMERO DE MESA ................................................................................................... VERSION DEL CUESTIONARIO DE EXAMEN ................................................................ Nº DE EXPEDIENTE ...................................................................................................... Nº DE D.N.I. O EQUIVALENTE PARA EXTRANJEROS ................................................... APELLIDOS Y NOMBRE ................................................................................................ ADVERTENCIAS: a) Escriba con BOLIGRAFO sobre superficie dura y lisa. b) No utilice lápiz, rotulador o pluma de tinta que se pueda borrar. c) Indique la respuesta que crea correcta en forma claramente legible. d) Las respuestas ilegibles o confusas o las indicadas con otra clase diferente de signos se penalizarán como incorrectas. e) Las equivocaciones deben subsanarse tachando íntegramente la respuesta errónea, sin que ésta quede visible, y escribiendo al lado la respuesta elegida. f) Si inutilizara esta hoja de respuesta devuelva los tres ejemplares de que se compone a la Mesa de Examen para recibir otra de repuesto y no olvide consignar sus datos personales.

1 2 3 4 5 6 7 8 9 10 11 12 13 14 15 16 17 18 19 20 21 22 23 24 25 26 27 28 29 30 31 32 33 34 35

E E B B C B B D A E B D E B D A E B B D A C E B A D D E C D E D D B E

36 37 38 39 40 41 42 43 44 45 46 47 48 49 50 51 52 53 54 55 56 57 58 59 60 61 62 63 64 65 66 67 68 69 70

D E D B D A D B D C E D C D B D B C B D B A B C C B D B D C B E C A D

71 72 73 74 75 76 77 78 79 80 81 82 83 84 85 86 87 88 89 90 91 92 93 94 95 96 97 98 99 100 101 102 103 104 105

C C E D C 0 A C B D A A E E E B A C D E E E E B B B A B D E C C A A E

106 107 108 109 110 111 112 113 114 115 116 117 118 119 120 121 122 123 124 125 126 127 128 129 130 131 132 133 134 135 136 137 138 139 140

D D E D D B A A 0 E A C A B D A B C C B D D B A D A A C B C D B B D B

141 142 143 144 145 146 147 148 149 150 151 152 153 154 155 156 157 158 159 160 161 162 163 164 165 166 167 168 169 170 171 172 173 174 175

B B C D A B D C A E A D E C E C C C B A B A C C A C C B A E B E E B B

176 177 178 179 180 181 182 183 184 185 186 187 188 189 190 191 192 193 194 195 196 197 198 199 200

A A C A D C E E C A D D E B B D A D D C E B C A C

View more...

Comments

Copyright ©2017 KUPDF Inc.
SUPPORT KUPDF